Performing Preparation, Compilation, and Review Engagements

106
Performing Preparation, Compilation, and Review Engagements Hugh Parker, CPA, Ph.D.; and Kimberly Burke, Ph.D. CPE/CPD = 2.2 credit hours

Transcript of Performing Preparation, Compilation, and Review Engagements

Page 1: Performing Preparation, Compilation, and Review Engagements

Performing Preparation, Compilation, and Review Engagements

Hugh Parker, CPA, Ph.D.; and Kimberly Burke, Ph.D.

CPE/CPD = 2.2 credit hours

PBV1701P

AICPAStore.com | CIMAglobal.com

1811-9880 Update CPE Text Covers w/ AICPA&CIMA logo.indd 1-3 11/30/18 3:49 PM

Page 2: Performing Preparation, Compilation, and Review Engagements
Page 3: Performing Preparation, Compilation, and Review Engagements

PERFORMING PREPARATION, COMPILATION, AND REVIEW

ENGAGEMENTS

BY HUGH PARKER, CPA, PH.D.; AND KIMBERLY BURKE, PH.D.

Page 4: Performing Preparation, Compilation, and Review Engagements

Notice to readers

Performing Preparation, Compilation, and Review Engagements is intended solely for use in

continuing professional education and not as a reference. It does not represent an official position

of the Association of International Certified Professional Accountants, and it is distributed with

the understanding that the author and publisher are not rendering legal, accounting, or other

professional services in the publication. This course is intended to be an overview of the topics

discussed within, and the author has made every attempt to verify the completeness and accuracy

of the information herein. However, neither the author nor publisher can guarantee the

applicability of the information found herein. If legal advice or other expert assistance is

required, the services of a competent professional should be sought.

Special note: COVID-19 resources

The Association, the global voice of the American Institute of CPAs and the Chartered Institute

of Management Accountants, is taking the Coronavirus (COVID-19) very seriously. We are

continually monitoring the virus’ impact on our members and the profession. For the most up-to-

date information on this topic, please visit the AICPA's Coronavirus Resource Center at

https://future.aicpa.org/resources/toolkit/aicpa-coronavirus-resource-center. For topic-specific

updates, please visit the following resource centers.

COVID-19 Resource Center Website

Audit and assurance https://future.aicpa.org/topic/audit-

assurance/covid-19-audit-assurance

Accounting & reporting https://future.aicpa.org/topic/accounting-

reporting/covid-19-accounting-reporting

Forensic services https://future.aicpa.org/topic/forensic-

services/covid-19-forensic-services

Valuation services https://future.aicpa.org/topic/valuation-

services/covid-19-valuation-services

Management accounting https://future.aicpa.org/topic/management-

accounting/covid-19-management-accounting

Personal financial planning https://future.aicpa.org/topic/financial-

planning/covid-19-personal-finance-planning

Small firm https://future.aicpa.org/topic/firm-practice-

management/covid-19-small-firms

Tax https://future.aicpa.org/topic/tax/covid-19-tax

Technology https://future.aicpa.org/topic/technology/covid-

19-technology

Government https://future.aicpa.org/topic/government/covid-

19-government

© 2021 Association of International Certified Professional Accountants, Inc. All rights reserved.

For information about the procedure for requesting permission to make copies of any part of this

work, please email [email protected] with your request. Otherwise,

requests should be written and mailed to Permissions Department, 220 Leigh Farm Road,

Durham, NC 27707-8110 USA.

CL2PCRV GS-0421-0A Revised: March 2021

Page 5: Performing Preparation, Compilation, and Review Engagements

Use of materials

This course manual accompanies all formats in which the course is offered, including self-study

text, self-study online, group study, in-firm, and other formats, as applicable. Specific

instructions for users of the various formats are included in this section.

CPAs are required to participate in continuing professional education (CPE) to maintain their

professional competence and provide quality professional services. CPAs are responsible for

complying with all applicable CPE requirements, rules, and regulations of state licensing bodies,

other governmental entities, membership associations, and other professional organizations or

bodies.

Professional standards for CPE programs are issued jointly by the American Institute of Certified

Public Accountants (AICPA) and the National Association of State Boards of Accountancy

(NASBA) to provide a framework for the development, presentation, measurement, and

reporting of CPE programs. The Statement on Standards for CPE Programs (CPE standards)

is available as part of AICPA Professional Standards, either in paperback or as an online

subscription through the Association’s Online Professional Library.

Review questions and exercises for self-study participants The CPE standards require that self-study programs include review questions/exercises that

provide feedback for both correct and incorrect responses. Note that these reviews are provided

only as learning aids and do not constitute a final examination.

Requirements for claiming and receiving CPE credit CPE standards place responsibility on both the individual participant and the program sponsor to

maintain a record of attendance at a CPE program. CPAs who participate in only part of a CPE

program, should claim CPE credit only for the portion that they attended or completed.

You must document your claims of CPE credit. Examples of acceptable evidence of completion

include:

For group and independent study programs, a certificate or other verification supplied by

the CPE program sponsor

For self-study programs, a certificate supplied by the CPE program sponsor after

satisfactory completion of an examination

When you participate in group study and other live presentations, you will receive a completion

certificate from the program sponsor. CPE program sponsors are required to keep documentation

on programs for five years, including records of participation.

When you participate in self-study, you must complete the exam within one year of the date of

course purchase to receive a certificate indicating satisfactory completion of the CPE program.

The exam for self-study in print format is located in the “Examination” section at the end

of the course manual.

Page 6: Performing Preparation, Compilation, and Review Engagements

You can find the course code number for both the self-study exam and the self-study evaluation in the examination’s introductory material. You will complete the self-study

exam and evaluation online at https://cpegrading.aicpa.org. You must provide the unique serial number printed on the inside front cover of this publication and you must achieve a minimum passing grade of at least 70 percent to qualify for CPE credit.

— Upon achieving a passing grade, you will receive a certificate displaying the number

of CPE credits earned based on a 50-minute learning segment, in compliance with

CPE standards. The grading system provides a completion certificate online, which

you may print or save as a PDF. The grading system maintains a transcript of your

completed courses.

— If you do not achieve a passing grade, the online grading system notifies you of this

and also provides instructions for retaking the exam. You have three attempts to pass

the exam. If you do not pass the exam in three attempts, please contact the Member

Service Center at 1.888.777.7077 to obtain additional attempts.

Program evaluations The information accumulated from participant evaluation forms is important in our continual

efforts to provide high quality continuing education for the profession. When you participate in

group study and other live presentations, please return your evaluation forms prior to departing

your program sessions. When you participate in self-study, please complete the course

evaluation online. Your comments are very important to us.

Customer service For help and support, including information on refund claims and complaint resolutions, please

call the Member Service Center at 1.888.777.7077, or visit the online help page at

https://future.aicpa.org/cpe-learning.

Page 7: Performing Preparation, Compilation, and Review Engagements

© 2021 Association of International Certified Professional Accountants. All rights reserved. Table of Contents 1

Table of Contents

Performing an Engagement to Prepare Financial Statements 1

Objective and scope 2

Performance requirements 4

Documentation for engagements to prepare financial statements 8

Summary 10

Practice questions 11

Solutions 13

Performing Compilation Engagements 15

Compilation framework and objectives 16

Compilation performance standards 21

Documentation for compilation engagements 24

Summary 28

Practice questions 29

Solutions 33

Performing Review Engagements 39

Review general principles and objective 40

Review performance standards 45

Documentation for review engagements 54

Change in level of service 58

Summary 59

Practice questions and case studies 60

Solutions 65

AR-C Glossary 71

Accounting and Auditing Glossary 77

Page 8: Performing Preparation, Compilation, and Review Engagements

© 2021 Association of International Certified Professional Accountants. All rights reserved. Table of Contents 2

Page 9: Performing Preparation, Compilation, and Review Engagements

© 2021 Association of International Certified Professional Accountants. All rights reserved. 1

Performing an Engagement to Prepare Financial Statements

Learning objectives

Identify the framework and objectives of engagements to prepare financial statements.

Recall the performance requirements for an engagement to prepare financial statements.

Recognize the minimum documentation requirements for an engagement to prepare financial statements.

Page 10: Performing Preparation, Compilation, and Review Engagements

© 2021 Association of International Certified Professional Accountants. All rights reserved. 2

Objective and scope The purpose of this section is to provide an overview of the nature of engagements to prepare financial

statements, types of financial information, and to distinguish between preparing and assisting in

preparing financial statements.

Definition

The objective of a preparation engagement is to prepare financial statements pursuant to a specified

financial reporting framework. That framework may be accounting principles generally accepted in the

United States of America (GAAP), International Financial Reporting Standards promulgated by the

International Accounting Standards Board, or a special purpose framework such as the cash- or tax-basis

of accounting.

Preparation of financial statements is a nonattest service, and as such differs significantly from reporting

services such as compilation, review, and audit. Because engagements to prepare financial statements

are nonattest services, the accountant is not required to consider whether they are independent. Further,

the accountant will not provide an opinion or conclusion on the financial statements—the objective is

simply to prepare them.

There are significant similarities between an engagement to prepare financial statements, which is a

nonattest service, and a compilation engagement, which is an attest service in which no assurance is

obtained or provided on the financial statements. One of these similarities is that there is no need to

gather supporting evidence in either engagement, nor is the accountant required to verify the information

provided by management. The significant differences between an engagement to prepare financial

statements and a compilation of financial statements are that (1) in a compilation of financial

statements, the accountant is required to determine whether he or she is independent of the entity, and

(2) engagements to prepare financial statements are not a reporting service, whereas a report is required

in a compilation of financial statements.

Some accountants may struggle to distinguish a preparation engagement conducted in accordance with

SSARS from bookkeeping services. The distinction lies in the intent of management. If management

engages the accountant to prepare financial statements, this is a preparation engagement to which

SSARS applies. If an accountant prepares financial statements as part of a bookkeeping or other service,

SSARS is not applicable. For example, if an accountant were hired by the client to perform bookkeeping

services and the client understands that the accounting software will automatically prepare financial

statements, SSARS does not apply.

An engagement to prepare financial statements can be undertaken for historical financial statements,

other historical financial information or prospective financial information, including

specified elements, accounts, or items of a financial statement. supplementary information.

Page 11: Performing Preparation, Compilation, and Review Engagements

© 2021 Association of International Certified Professional Accountants. All rights reserved. 3

required supplementary information. pro forma financial information.

Professional judgment

Professional judgment is exercised by an accountant whose training, knowledge, and experience have

assisted in developing the necessary competencies to achieve reasonable judgments and make

informed decisions about appropriate courses of action when undertaking an engagement in accordance

with Statements on Standards for Accounting and Review Services (SSARSs). Professional judgment is

essential to the conduct of an engagement to prepare financial statements because accountants who

prepare financial statements are often required to interpret accounting information, SSARSs, and ethical

information based on their knowledge of and experience with the facts and circumstances, including

knowledge acquired from engagements carried out for the client’s financial statements in prior periods.

the accountant’s understanding of the entity and its environment, including its accounting system, and of the application of the applicable financial reporting framework in the client’s industry.

the extent to which the preparation and presentation of the financial statements requires the exercise of judgment by management or the accountant.

The best measure of professional judgment is often a post hoc evaluation of the degree to which

decisions made and conclusions reached are consistent with the guidance provided for the engagement,

and are reasonable based on the information available to the accountant at the time the decision was

made or conclusions reached. Specifically, for an engagement to prepare financial statements, the best

measure of professional judgment would include an evaluation of the prepared financial statements

considering the information available at the time the statements were prepared.

Knowledge check

1. An accountant is not required to comply with AR-C section 70, Preparation of Financial Statements,1 in an engagement to prepare

a. A single balance sheet prepared in accordance with an SPF. b. An adjusting journal entry to record depreciation. c. A full set of financial statements prepared in accordance with GAAP. d. A schedule of required supplementary information.

1 All AR-C sections can be found in AICPA Professional Standards.

Page 12: Performing Preparation, Compilation, and Review Engagements

© 2021 Association of International Certified Professional Accountants. All rights reserved. 4

Performance requirements The following are the performance requirements set forth in AR-C section 70, with respect to which an

accountant is required to comply in an engagement to prepare financial statements.

Perhaps what is most notable about these requirements, particularly when compared to requirements for

compilation, review, or audit engagements, is that there is no reporting requirement. In addition, unlike the

performance requirements for review and audit engagements, there are no requirements for the

accountant to perform evidence-gathering procedures such as analytical procedures, inquiries, tests of

controls, transactions, or balances.

There are three primary performance requirements for a preparation engagement, each of which is

discussed in subsequent sections:

Agree upon the terms of the engagement with the client. Obtain an understanding of the financial reporting framework and significant accounting policies

intended to be used in the preparation of financial statements. Prepare the financial statements.

Agree upon the terms of the engagement with the client

For engagements to prepare financial statements, the accountant should agree upon the terms of the

engagement with the client in writing. The written agreement is most commonly documented in the form

of an engagement letter, but a formal contract is also suitable. An oral understanding of the terms of the

engagement is not enough to meet the requirement. We have long considered engagement letters to be

a practical and prudent best practice, even before they were required for SSARSs engagements. The

written engagement letter helps avoid client misunderstandings, clearly indicates the scope of the

engagement, reduces the potential legal liability, improves practice management, and clarifies the

contractual obligation between accountant and client. Although the performance standards for an

engagement to prepare financial statements require the accountant to obtain an understanding of (1) the

financial reporting framework and (2) the significant accounting policies intended to be used in the

preparation of the financial statements in order to perform the engagement, the accountant should have

some level of understanding prior to determining the scope of the engagement.

Typically, a new engagement letter would be obtained annually, covering the services to be performed.

We believe that this is the best way to manage the risk that the understanding with the client has

diminished over time or that the engagement is seen as continuous from a legal perspective.

Under some circumstances, a third party may contract for an engagement to prepare an entity’s financial

statements. In that case, although the third party may sign the engagement letter, the accountant is still

required to obtain management’s agreement to the terms of the engagement and would need

management to sign the engagement letter.

Page 13: Performing Preparation, Compilation, and Review Engagements

© 2021 Association of International Certified Professional Accountants. All rights reserved. 5

SSARSs also require that the engagement letter or other written agreement be signed by both the

accountant (or the accountant’s firm) and management or those charged with governance, as

appropriate.

Specifically, the agreed-upon terms of the engagement to prepare financial statements should include

the following:

The objective of the engagement to prepare financial statements in accordance with a financial reporting framework

The responsibilities of management as noted in the previous discussion of the requirements of AR-C section 60, General Principles for Engagements Performed in Accordance With Statements on Standards for Accounting and Review Services

The agreement of management that each page of the financial statements will include a statement indicating that no assurance is provided on the financial statements, or the accountant will be required to issue a disclaimer that makes clear that no assurance is provided on the financial statements

The responsibilities of the accountant The limitations of the engagement Identification of the applicable financial reporting framework for the preparation of financial

statements Whether the financial statements are to contain a known departure or departures from the applicable

financial reporting framework (including inadequate disclosure) or omit substantially all disclosures required by the applicable financial reporting framework

Other matters that may be addressed in the engagement letter include

fees and billings; and any limitation of, or other arrangements regarding the liability of, the accountant or the client, such as

indemnification to the accountant for liability arising from the knowledge of misrepresentations made to the accountant by management.

Knowledge and understanding of the client’s financial reporting framework

Accountants should obtain knowledge and understanding of the client’s financial reporting framework

and significant accounting policies intended to be used in preparing the financial statements. The

accountant does not have to have this knowledge at the time the engagement is accepted, if that

knowledge is obtained before the engagement is completed. Most often, knowledge of the client’s

financial reporting framework is gained through inquiry and experience with the client or experience with

other entities in the same industry as the client. Enough understanding of the client’s financial reporting

framework and significant accounting policies intended to be used in preparing the financial statements

can be obtained by consulting AICPA guides, industry publications, textbooks, periodicals, appropriate

continuing professional education, or individuals knowledgeable about the industry.

Page 14: Performing Preparation, Compilation, and Review Engagements

© 2021 Association of International Certified Professional Accountants. All rights reserved. 6

Knowledge check

2. Which is not a performance requirement for an engagement to prepare financial statements?

a. Obtain an understanding of the client’s industry. b. Prepare the financial statements. c. Agree upon the terms of the engagement with the client. d. Obtain an understanding of the financial reporting framework.

Preparing the financial statements

The accountant should prepare the financial statements using his or her understanding of the applicable

financial reporting framework and the records, documents, explanations, and other information provided

by management. Note that although no evidence-gathering procedures are required of the accountant, it

may be necessary to inquire of management regarding assumptions underlying the financial statements.

Statement of no assurance

Each page of the prepared financial statements, including the related notes, should include a statement

that indicates that, at a minimum, no assurance has been provided on the financial statements. This

statement is intended to avoid misunderstandings on the part of users with respect to the accountant’s

involvement with the financial statements. The form of the statement is up to management’s discretion if

the statement is not misleading; the accountant’s firm name is not required to be included. Examples of

statements include the following:

No assurance is provided on these financial statements. These financial statements have not been subjected to an audit or review or compilation

engagement, and no assurance is provided on them.

If the accountant is unable to include a statement on each page of the financial statements, he or she

should either issue a disclaimer, perform a compilation engagement in accordance with AR-C section 80,

Compilation Engagements, or withdraw from the engagement. A disclaimer might read as follows:

The accompanying financial statements of XYZ Company as of and for the year ended December

31, 20X2, were not subject to an audit, review, or compilation engagement by me (us) and,

accordingly, I (we) do no express an opinion, a conclusion, nor provide any assurance on them.

Accountants who choose to withdraw from an engagement must inform management of their reasons

for withdrawing. Application guidance also suggests that accountants may want to consult with legal

counsel before withdrawing from an engagement.

Special issues

Financial statements prepared in accordance with an SPF, such as the cash-, tax-, modified cash-,

regulatory-, or contractual-basis of accounting require some additional information. For these financial

statements, accountants should disclose a description of the financial reporting framework. This

Page 15: Performing Preparation, Compilation, and Review Engagements

© 2021 Association of International Certified Professional Accountants. All rights reserved. 7

disclosure may be made on the face of the financial statements or in a note to the financial statements.

Often, that disclosure is made next to or under the title of the financial statements (for example,

“Statements of Assets and Liabilities—Modified Cash-Basis”).

Accountants may prepare financial statements that omit substantially all disclosures required by the

financial reporting framework under certain circumstances. If management elects to omit substantially

all disclosures required by the financial reporting framework this should be disclosed on the face of the

financials or in a selected note to the financial statements. Specifically, the accountant may do so if he or

she does not have reason to believe that the omission was undertaken with the intention of misleading

users of the financial statements. When the accountant prepares financial statements that include

disclosures about only a few matters in the notes to the financial statement, those disclosures may be

labeled “Selected Information—Substantially All Disclosures Required by [the applicable financial reporting

framework] Are Not Included.”

Given the nature of an engagement to prepare financial statements, it would not be unusual for the client to

ask for the accountant’s assistance with significant judgments regarding amounts or disclosures to be

reflected in the financial statements. On those occasions, the accountant can use his or her expertise to

assist management in deriving those significant judgments (for example, advising on alternative

accounting policies or helping with material estimates). The accountant is required to discuss those

judgments with management so that management understands them and accepts responsibility for them.

Concerns

Two issues may arise that confound the engagement to prepare financial statements. The first occurs if

the accountant becomes aware that the records, documents, explanations, or other information

(including significant judgments used in the preparation of financial statements) are incomplete,

inaccurate, or otherwise unsatisfactory. This judgment may be based on knowledge gained during the

engagement, knowledge gained from other engagements, or other sources.

In these circumstances, the accountant should bring the matter to the attention of management and

request additional or corrected information. In obtaining additional or corrected information, the

accountant may make inquiries of management or review documents that may be considered more

appropriate for a reporting engagement. (Note that the accountant is not required to verify, corroborate,

or substantiate this additional or corrected information.) Although these procedures exceed the typical

performance requirements of the engagement to prepare financial statements, they do not change the

nature of the engagement.

The second issue may arise if the accountant, after discussion with management, prepares financial

statements that contain a known departure or departures from the applicable financial reporting

framework (including inadequate disclosure). For example, the financial statements are prepared in

accordance with GAAP, but depreciation is calculated using tax methods. Ultimately, the accountant

should disclose the material misstatement(s) in the financial statements. This disclosure may be made

on the face of the financial statements or in a note to the financial statements.

Page 16: Performing Preparation, Compilation, and Review Engagements

© 2021 Association of International Certified Professional Accountants. All rights reserved. 8

Documentation for engagements to prepare financial statements Documentation is necessary to support compliance with the standards that govern engagements to

prepare financial statements and the firm’s quality control procedures. The extent of documentation will

vary depending on the circumstances of the engagement, the methodology and tools used, as well as on

the accountant’s professional judgment. The following discussion first identifies the minimum

documentation requirements included in SSARSs. Accountants, however, are not precluded from

exceeding these minimum requirements and including, for example, written documentation contained in

other engagement files or quality control files. In addition, we have provided a discussion of the

additional documentation items we believe are necessary to support compliance with standards for an

engagement to prepare financial statements.

Required documentation

According to AR-C section 70, documentation for engagements to prepare financial statements should

include the following:

The engagement letter, or other suitable written form of written documentation with management A copy of the financial statements that the accountant has prepared

Client acceptance form

Although not required, we recommend the use of a client acceptance form that can be filed in the

permanent file for the client. The form, which we recommend updating at least annually, notes the

relevant industry and associated industry guides for the client and answers the basic questions about the

client’s business that will be useful in preparing the financial statements. The form can also be used to

identify financial statement captions that will require special attention or adjustments, as well as

document information concerning unique accounting principles and practices, related parties, and so on.

It is beneficial to cross-reference this form to the disclosure checklist and requisite report modifications.

Disclosure checklist

Unless an explicit decision is made by management (not the accountant) to omit substantially all

disclosures from the prepared financial statements, all disclosures required by the applicable financial

reporting framework must be included with the financial statements. Because the list of possible

disclosures is very long, and the risk of inadvertently omitting a required disclosure can be high, the use

of a relevant disclosure checklist is advisable.

Page 17: Performing Preparation, Compilation, and Review Engagements

© 2021 Association of International Certified Professional Accountants. All rights reserved. 9

Knowledge check

3. Which option correctly describes the required documentation for a preparation engagement?

a. Signed engagement letter, financial statements, preparation report. b. Signed engagement letter, prepared financial statements. c. Signed engagement letter, prepared financial statements, management representation letter,

preparation report. d. Signed engagement letter, prepared financial statements, disclosure checklist.

Page 18: Performing Preparation, Compilation, and Review Engagements

© 2021 Association of International Certified Professional Accountants. All rights reserved. 10

Summary We have just discussed the following points:

Definition of an engagement to prepare financial statements Performance requirements for engagements to prepare financial statements Knowledge and understanding of the client’s financial reporting framework Preparation of the financial statements Minimum documentation requirements for an engagement to prepare financial statements Forms, checklists, and other documentation in an engagement to prepare financial statements

Page 19: Performing Preparation, Compilation, and Review Engagements

© 2021 Association of International Certified Professional Accountants. All rights reserved. 11

Practice questions 1. For each of the following independent situations, determine whether AR-C section 70 applies.

a. The client engages accountant A to prepare financial statements for an upcoming review of those financial statements by accountant B.

Applies ______________ Does Not Apply ____________

b. The client engages accountant A to review financial statements. Prior to conducting the review engagement, accountant A prepares the financial statements.

Applies ______________ Does Not Apply ____________

c. Accountant A proposes adjustments for deferred income taxes.

Applies ______________ Does Not Apply ____________

d. Accountant A prepares a balance sheet with substantially all disclosures omitted for the client.

Applies ______________ Does Not Apply ____________

e. Accountant A drafts the financial statement notes for the client.

Applies ______________ Does Not Apply ____________

Case: Preparation engagements

Connery, Moore, and Brosnan (CMB), LLC is a public accounting firm with a significant number of clients for whom it provides accounting and compilation services. CMB was recently engaged by OddJobs Corp, a residential refurbishment company, to prepare its financial statements. OddJobs does not need a compilation report on the financial statements. The president and CEO of OddJobs is Mr. Ernie Blofeld.

Because this is the first client to request the firm to prepare financial statements that would not be subject to a compilation, CMB assigned one of the younger partners, Craig Daniels, to oversee the engagement. Daniels met with Ernie Blofeld several times to discuss the engagement and explained that CMB would not issue a report on the financial statements, and that each page of the financial statements would include a legend that indicated no assurance had been provided. Blofeld agreed and signed a standard engagement letter that included the following language with respect to CMB’s responsibilities:

We are not required to, and will not, verify the accuracy or completeness of the information you will provide to us for the engagement or otherwise gather evidence for expressing an opinion or a conclusion. Accordingly, we will not express an opinion or a conclusion or provide any assurance on the financial statements.

Page 20: Performing Preparation, Compilation, and Review Engagements

© 2021 Association of International Certified Professional Accountants. All rights reserved. 12

Case: Preparation engagements (continued)

You agree that we will prepare the financial statements to clearly indicate that no

assurance has been provided on them.

After CMB prepared the financial statements using OddJobs’s accounting software, it met with Blofeld and his management team. Each page of the financial statements included the statement, “No assurance is provided on these financial statements.” At that time, Blofeld noted his concern that the company’s bank, which is used to receiving financial statements with a compilation report attached, would not understand the legend, and its business may be adversely affected.

Required

1. Assume that Blofeld remains convinced that the financial statements with the legend stating that “no assurance is provided” on them will not be acceptable for OddJobs’s bank unless accompanied by an accountant’s report. What alternatives does CMB have?

2. According to SSARS, does CMB have an obligation to ensure that the financial statements are not provided to a third party without the legend or a disclaimer?

3. What should CMB do if it becomes concerned that the financial statements have been provided to a third party without the legend or disclaimer?

Page 21: Performing Preparation, Compilation, and Review Engagements

© 2021 Association of International Certified Professional Accountants. All rights reserved. 13

Solutions

Performing an Engagement to Prepare Financial Statements

Knowledge check solutions

1.

a. Incorrect. An engagement to prepare a single financial statement would be performed in accordance with AR-C section 70.

b. Correct. Assisting the client with an adjusting journal entry does not constitute an engagement to prepare financial statements.

c. Incorrect. An engagement to prepare a full set of financial statements would be performed in accordance with AR-C section 70.

d. Incorrect. An engagement to prepare required supplementary information would be performed in accordance with AR-C section 70.

2.

a. Correct. The accountant is not required to obtain an understanding of the client’s industry for a preparation engagement.

b. Incorrect. The accountant is required to prepare the financial statements in an engagement to prepare financial statements.

c. Incorrect. The accountant is required to agree upon the terms of the engagement with the client in an engagement to prepare financial statements.

d. Incorrect. The accountant is required to obtain an understanding of the financial reporting framework and significant accounting policies.

3.

a. Incorrect. A report is not required for a preparation engagement.

b. Correct. A preparation engagement’s documentation should include a signed engagement letter and prepared financial statements.

c. Incorrect. A preparation engagement does not require a management representation letter or a report.

d. Incorrect. An engagement to prepare financial statements does not require a disclosure checklist for documentation, although the authors recommend its use when substantially all disclosures are not omitted.

Page 22: Performing Preparation, Compilation, and Review Engagements

© 2021 Association of International Certified Professional Accountants. All rights reserved. 14

Practice question solutions

1.

a. Applies ______X______ Does Not Apply ____________

b. Applies ____________ Does Not Apply ______X______

c. Applies ____________ Does Not Apply ______X______

d. Applies ______X______ Does Not Apply ____________

e. Applies ______X______ Does Not Apply ____________

Case: Preparation engagements solutions

1.

CMB could have a conversation with Mr. Blofeld about performing a compilation service with respect to the financial statements that CMB prepared. CMB would need to obtain a new engagement letter for the compilation service. After performance of the required compilation procedures, CMB would issue the appropriate compilation report.

2.

SSARSs require that the client agrees in the engagement letter to provide an indication that no assurance is provided on the financial statements. The accountant is not responsible for controlling distribution of the financial statements once such financial statements are provided to the client.

3.

If CMB believes that the financial statements have been subsequently provided to a third party without the legend or disclaimer, CMB is in a difficult business situation.

The issue in this situation becomes one of the risk management for CMB. CMB could contact the bank directly and inform them that the financial statements should include this legend or disclaimer. However, CMB is not certain that the financial statements have been provided to the bank without the legend, and providing inaccurate information to the bank can hurt CMB’s credibility. In addition, if the bank were to react punitively toward Blofeld, Blofeld might sue CMB for unnecessarily damaging its business, arguing that the business is unaffected by the presentation of these financial statements. Finally, by providing this information without Blofeld’s consent, CMB would be in violation of the confidentiality rule of the AICPA Professional Code of Conduct.

This question points to two important business considerations for CPAs. First, CPAs should carefully consider the provisions of their liability insurance. Given these circumstances, the CPA could contact his or her insurance carrier for consultation and advice. Some insurers make it riskless for the CPA to ask for consultation; others may not. Therefore, CPAs should investigate the benefits and constraints provided by different insurance carriers before purchasing accountant’s liability insurance. Second, CPAs should establish a relationship with legal counsel before an issue like this arises. Legal counsel should be familiar with the laws and regulations governing CPAs in the states in which they practice.

Page 23: Performing Preparation, Compilation, and Review Engagements

© 2021 Association of International Certified Professional Accountants. All rights reserved. 15

Performing Compilation Engagements

Learning objectives

Identify the objective and framework of compilation engagements.

Recognize the elements that make up compilation engagements.

Distinguish compilation engagements from preparation engagements.

Identify the performance requirements for a compilation engagement.

Recognize the documentation requirements for a compilation engagement.

Page 24: Performing Preparation, Compilation, and Review Engagements

© 2021 Association of International Certified Professional Accountants. All rights reserved. 16

Compilation framework and objectives This section provides an overview of the objective and framework of compilation engagements and how

they compare to other engagements such as preparation, review, or audit.

Definition

The objective of the accountant in a compilation engagement is to apply accounting and financial

reporting expertise to assist management in the presentation of financial statements and report in

accordance with this section without undertaking to obtain or provide any assurance that there are no

material modifications that should be made to the financial statements in order for them to be in

accordance with the applicable financial reporting framework. Because compiled financial statements

provide the reader no assurance regarding the statements, they represent one of the lowest levels of

financial statement services accountants can provide to their clients.

One of the most significant changes that affects compilation engagements that was brought about by

the issuance of Statements on Standards for Accounting and Review Services (SSARSs) No. 21,

Statements on Standards for Accounting and Review Services: Clarification and Recodification (AICPA,

Professional Standards), was that under the new standard, accountants are required to undertake a

compilation engagement only whenever they are engaged to do so. Until very recently, accountants were

also required to perform a compilation engagement whenever they prepared and presented financial

statements to the client or third parties.

Compilation engagements are attest (not assurance) services that may be provided only to nonpublic

entities, referred to in the literature as non-issuers. Accountants may not perform compilations for

entities that have listed securities or filings with a regulatory agency in preparation for a public sale of

securities or a subsidiary, or who are joint ventures or controlled enterprises of a public entity.

Elements of a compilation engagement

Any compilation engagement involves four common elements:

A three-party relationship involving management, an accountant, and intended users An applicable financial reporting framework Financial statements or financial information A written communication or report

Three-party relationship

The three-party relationship includes management or a responsible party, an accountant in the practice

of public accounting (as defined by the AICPA Code of Professional Conduct), and the intended users of

the financial statements or financial information. Although management and the intended users may

often be the same, intended users may be from different entities, like a bank or potential investor.

Page 25: Performing Preparation, Compilation, and Review Engagements

© 2021 Association of International Certified Professional Accountants. All rights reserved. 17

In this three-party relationship, management is responsible for the selection of the applicable financial

reporting framework and the preparation and fair presentation of the financial statements in accordance

with the applicable financial reporting framework. These management responsibilities do not preclude

the accountant from making suggestions about the financial statements or even preparing them, in

whole or in part, based on information provided by management. If, however, management is unwilling to

accept its responsibilities, the accountant may not issue an unmodified compilation report.

Accountants undertaking a compilation engagement should possess an appropriate level of knowledge

of an entity’s financial reporting framework to assist management in the presentation of financial

statements that are appropriate in form for the entity.

Intended users of the financial statements are those persons who understand the limitations of the

compilation engagement and the financial statements. Although the intended users of the financial

statements are an integral part of the three-party relationships found in any compilation engagement, the

accountant has no obligation to identify the intended users.

On occasion, intended users, such as bankers or regulators, may require or ask the client to request that

additional procedures be performed in conjunction with the compilation engagement. For example, an

intended user may ask that certain agreed-upon procedures, such as a review of account reconciliations,

be performed. Accountants may perform these additional services in conjunction with the compilation

engagement as long as the accountant adheres to professional standards regarding those services.

Applicable financial reporting framework

A financial reporting framework encompasses financial accounting standards established by an authorized

or recognized standards-setting organization. The requirements of the applicable financial reporting

framework determine the form and content of the financial statements. Financial reporting frameworks

include both U.S. generally accepted accounting principles (GAAP) and special purposes frameworks

(SPFs). GAAP is promulgated by FASB, GASB, or the Federal Accounting Standard Advisory Board. SPFs

include other comprehensive bases of accounting (OCBOA), the contractual-basis of accounting, and

International Financial Reporting Standards issued by the International Accounting Standards Board.

Management, or those charged with governance, is ultimately responsible for selecting the entity’s

applicable financial reporting framework as well as choosing among individual accounting policies when

the framework provides acceptable alternatives.

Financial statement or financial information

Accountants may be engaged to perform compilation engagements on complete sets of financial

statements, individual financial statements, or specified elements, accounts, or items of a financial

statement. Accountants may also be engaged to perform compilation engagements on supplementary

information, required supplementary information, other historical financial information, pro forma

financial information, or prospective financial information.

Page 26: Performing Preparation, Compilation, and Review Engagements

© 2021 Association of International Certified Professional Accountants. All rights reserved. 18

A written communication or report

Accountants performing compilation engagements should provide a written report unless the accountant

withdraws from the engagement. Note that if an accountant is not independent, he or she may issue a

compilation report provided the accountant complies with the compilation standards.

Comparison to other engagements

This section provides a brief comparison of compilation engagements to other engagements related to

the financial statements that may be provided to nonpublic entities. These services range from

preparation services to review or audit engagements.

Accountants may report on financial statements after conducting an audit, review or compilation

engagement. The audit engagement provides the highest level of assurance (reasonable assurance) that

the financial statements are free of material misstatement and requires the auditor to gather significant

evidence in order to reach this conclusion. The review engagement requires less evidence and fewer

procedures to achieve a lower level of assurance (limited assurance) that there are no material

modifications that should be made to the financial statements. The compilation engagement offers a

significantly lower level of service than the review in that no assurance is provided as a result of a

compilation engagement. Although a compilation is an attest service like audit or review, it does not

provide an appropriate basis for obtaining any assurance on the financial statements.

After years of only allowing compilation or review engagements, in 2014, the Accounting and Review

Services Committee (ARSC) issued a standard that allows accountants to prepare financial statements

for their clients. The preparation service is a nonattest engagement, and it provides no assurance or even

a report on the financial statements. It is tempting to see the preparation and compilation engagements

as being the same, because neither provides assurance on the financial statements. It is important,

however, to understand the difference. Because the preparation service is a nonattest service,

accountants who prepare financial statements do not have to consider whether they are independent.

However, for compilation engagements, independence must be considered and disclosed if it is lacking.

Also, the preparation service does not require a report, although the accountant is not precluded from

providing one. The compilation engagement does require a report to accompany the financial

statements, and the performance requirements for the accountant providing a compilation engagement

exceed the performance requirements for an accountant providing a preparation service.

A word about the distinction between preparation and compilation engagements is in order. Historically,

compilation engagements have taken more than one form. One version of a compilation engagement

has been the “read and report” compilation in which the client prepares its financial statements and asks

the accountant to perform a compilation engagement on those prepared financial statements. What has

been perhaps more common is a request from the client for help in preparing financial statements. Until

October 2014, the accountant could assist in the preparation of the financial statements, but was also

required to perform a compilation engagement resulting in the issuance of a compilation report on those

Page 27: Performing Preparation, Compilation, and Review Engagements

© 2021 Association of International Certified Professional Accountants. All rights reserved. 19

financial statements. With the issuance of AR-C section 70, Preparation of Financial Statements1,

accountants are not required to perform a compilation engagement when the client wants assistance

only in preparing its financial statements. Under the current guidance, a client can engage its accountant

to perform a compilation engagement that is either a “read and report” compilation engagement or an

engagement where the accountant assists in preparing the financial statements and then reports on

them. The difference is that if the client seeks help only in preparing the financial statements and does

not want a compilation engagement or report, the accountant can perform a preparation service rather

than a compilation engagement. Thus, the client’s needs determine what kind of engagement

(preparation or compilation) the accountant performs.

The fundamental change to an engagement-basis is significant. Clients may choose whether they want

to engage accountants to

prepare financial statements, prepare financial statements and perform a compilation engagement, or read and report (compilation) on financial statements prepared by the client.

The prior authoritative literature did not allow accountants to be associated with financial statements

unless there was a compilation, review, or audit report on those financial statements. Accountants who

have provided compilation services on financial statements that have required an associated report (for

example, those that were not management-use-only) may now prepare financial statements without an

associated report.

The following table illustrates the similarities and differences among compilation engagements under the

previous SSARSs, compilation engagements under the new standards, and preparation engagements.

Under previous SSARSs Current SSARSs

Compilation Compilation Preparation

When does the standard apply?

When an accountant 1. is engaged to perform a compilation, or 2. submits financial statements to the client or third parties.

2

When an accountant is engaged to perform a compilation.

When an accountant is engaged to prepare financial statements.

Is an engagement letter required?

3 Yes. Yes. Yes.

1 All AR-C sections can be found in AICPA Professional Standards.

2 Submission of financial statements is defined as presenting financial statements that the accountant has

prepared to the client or third parties. 3 The accountant may obtain one engagement letter that covers all services performed.

Page 28: Performing Preparation, Compilation, and Review Engagements

© 2021 Association of International Certified Professional Accountants. All rights reserved. 20

Under previous SSARSs Current SSARSs

Compilation Compilation Preparation

Is the accountant required to determine if he or she is independent of the client?

Yes. Yes. No.

If the accountant is not independent, is that fact required to be disclosed?

Yes. Yes. N/A.

Does the engagement require a report?

Yes, unless the financial statements are management-use-only.

Yes. No4

May the financial statements go to users outside of management?

Yes, unless the financial statements are management-use-only.

Yes. Yes.

May the financial statements omit notes?

Yes. Yes. Yes.

Knowledge check

1. Which is not an element of a compilation engagement?

a. Three-party relationship. b. Providing assurance on financial statements. c. An applicable financial reporting framework. d. A written communication or report.

4 If the accountant is unable to have an adequate statement on each page of the financial statements that indicates

no assurance has been provided on those statements, the accountant is required to issue a disclaimer on the

financial statements.

Page 29: Performing Preparation, Compilation, and Review Engagements

© 2021 Association of International Certified Professional Accountants. All rights reserved. 21

Compilation performance standards The performance standards for compilation engagements include the following, each of which is

discussed in the subsequent sections:

Establish an understanding with the client Have appropriate knowledge and understanding of the entity’s financial reporting framework Read the financial statements Obtain additional information if the information supplied is incorrect, incomplete, or otherwise

unsatisfactory

Generally speaking, performance standards for any engagement are designed to enable the accountant

to achieve the appropriate level of assurance that there are no material misstatements in the financial

statements:

In order to achieve reasonable assurance in an audit, several performance standards must be met, including obtaining an understanding of the entity and its internal controls and gathering sufficient evidence to support the auditor’s conclusions.

In order to achieve limited assurance in a review engagement, there are fewer performance requirements than in an audit, but the accountant should subject the financial statements to inquiry and analytical procedures in order to support his or her conclusions.

However, in a compilation engagement, in which there is no assurance provided, the performance

requirements are quite limited. In fact, compliance with the compilation performance standards is not

enough to enable the accountant to express any degree of assurance. Note that accountants are

required to comply with these performance standards discussed in the following sections whenever they

have been engaged to compile financial statements.

Establish an understanding

Accountants performing compilation services are required to comply with AR-C section 60, General

Principles for Engagements Performed in Accordance with Statements on Standards for Accounting and

Review Services. As such, accountants must follow the guidance regarding the pre-engagement

decisions to accept or continue a client. In addition, once the decision is made to accept or continue the

client, accountants performing compilation services must also follow the guidance regarding a written,

signed agreement between the client and accountant. The written agreement is typically in the form of an

engagement letter, but a formal contract is also suitable. An oral understanding of the terms of the

engagement is not enough to meet the requirement. Although the performance standards for an

engagement to compile financial statements require the accountant to have knowledge and

understanding of the financial reporting framework and the significant accounting policies adopted by

management in order to perform the engagement, the accountant should have some level of

understanding prior to determining the scope of the engagement.

Page 30: Performing Preparation, Compilation, and Review Engagements

© 2021 Association of International Certified Professional Accountants. All rights reserved. 22

Knowledge and understanding of the entity’s financial reporting framework

Accountants are required to obtain an understanding of the applicable financial reporting framework

intended to be used in preparation of the financial statements and the significant accounting policies

adopted by management. The requirement to obtain this information does not prevent the accountant

from accepting a compilation engagement for an entity in an industry in which the accountant has no

previous experience. The accountant may obtain such understanding, for example, by consulting AICPA

guides, industry publications, financial statements of other entities in the industry, textbooks and

periodicals, appropriate continuing professional education, or individuals who are knowledgeable about

the framework or the industry.

Read the financial statements

The accountant should read the financial statements in light of the accountant’s understanding of the

applicable financial reporting framework and the significant accounting policies adopted by management

and consider whether the financial statements appear to be appropriate in form and free from obvious

material misstatements. In this context, the term errors refers to mistakes in the preparation of the

financial statements, including arithmetical or clerical mistakes, and mistakes in the application of

accounting principles, including inadequate disclosure.

The reading of the financial statements should be performed toward the end of the engagement, when

the accountant is almost ready to present the financial statements to the client.

Obtain additional information

If the accountant becomes aware that the information supplied is incorrect, incomplete, or otherwise

unsatisfactory, or that fraud or an illegal act may have occurred, he or she should bring that to the

attention of management and request additional or revised information. This judgment is based on the

financial statements and the adjusted trial balance and knowledge gained during the compilation

engagement and from prior engagements.

The accountant may become aware that the financial statements do not adequately refer to or describe

the applicable financial reporting framework (either in the titles of the statements or in the notes to the

statements).The accountant should identify the appropriate revisions to the financial statements that are

required in order for them to be presented in accordance with the applicable financial reporting

framework. Alternatively, the accountant may conclude that the financial statements are misleading.

In these circumstances, the accountant should bring these matters to the attention of management or

those charged with governance and request that management consider the effect of these matters on

the financial statements and communicate the results of their consideration to the accountant.

Page 31: Performing Preparation, Compilation, and Review Engagements

© 2021 Association of International Certified Professional Accountants. All rights reserved. 23

Additionally, the accountant should consider the effect of management’s conclusions regarding these

matters on the compilation report. In obtaining additional or revised information, the accountant may

perform procedures not ordinarily required for a compilation engagement. For example, the accountant

may make inquiries of management or review an account reconciliation to obtain additional or revised

information. (Note, however, that the accountant is not required to verify, corroborate, or substantiate this

additional information.) Although these procedures exceed the performance requirements of the

compilation, they do not change the nature of the engagement to a higher level of assurance. These

procedures should be documented in the working papers, but no reference is made to them in the

compilation report.

Ultimately, if the accountant believes the financial statements may be materially misstated, he or she

should obtain additional or revised information. The accountant should withdraw from the engagement

and inform management of the reasons for withdrawing if (1) management fails to provide records,

documents, explanations, or other information, including significant judgments, as requested; or

(2) management does not make appropriate revisions that are proposed by the accountant or does not

disclose such departures in the financial statements, and the accountant determines to not disclose

such departures in the accountant’s compilation report.

Fraud or illegal acts

The information acquired by the accountant during a compilation may cause the accountant to be

concerned about the possibility of fraud or illegal acts. Neither the SSARSs requirements nor application

literature specifically address the accountant’s responsibilities related to fraud or illegal acts. Although

accountants who perform compilation engagements are under no obligation to search for fraud or illegal

acts, we recommend communication between the accountant and management or those charged with

governance when circumstances indicate the presence of either.

Knowledge check

2. When evidence arises that information supplied is incorrect, incomplete, or otherwise unsatisfactory, what should an accountant do?

a. Bring that to the attention of management and request additional or revised information. b. Issue a qualified opinion. c. Issue a disclaimer on the compiled financial statements. d. Withdraw from the engagement.

Page 32: Performing Preparation, Compilation, and Review Engagements

© 2021 Association of International Certified Professional Accountants. All rights reserved. 24

Documentation for compilation engagements Documentation is necessary to support compliance with the standards that govern compilation

engagements and the firm’s quality control procedures, as well as to minimize liability. The extent of

documentation will vary depending on the circumstances of the engagement, the methodology and tools

used, and on the accountant’s professional judgment. The following discussion first identifies the

minimum documentation requirements included in SSARSs. Accountants, however, are not precluded

from exceeding these minimum requirements and including, for example, written documentation

contained in other engagement files or quality control files. In addition, we have provided a discussion of

the additional documentation items we believe are necessary to support compliance with standards for

compilation engagements.

Required documentation

According to AR-C section 80, Compilation Engagements, documentation for compilation engagements

should include the following:

The signed, written engagement letter documenting the understanding with the client. This letter must be signed by both management and the accountant.

A copy of the financial statements A copy of the accountant’s report

Documentation of knowledge and understanding of an entity’s financial reporting framework

Typically, documentation of the accountant’s knowledge and understanding of an entity’s financial

reporting framework is captured in a client acceptance form that can be filed in a permanent file or the

latest working paper file. The form, which should be updated at least annually, notes the relevant industry

and associated industry guides for the client and answers the basic questions about the client business.

A client acceptance form highlights information similar to that found on an audit planning form. The use

of a risk approach for a compilation engagement helps identify financial statement captions that will

require special attention or adjustments and helps focus the documentation on critical considerations.

However, it is important to note that no risk assessment procedures like those required in an audit are

required for compilation engagements. That said, the risk approach is a helpful tool practitioners can use

to maximize both efficiency and effectiveness. Information concerning unique accounting principles and

practices, related parties, and so on, should be cross-referenced to the disclosure checklist and requisite

report modifications.

Page 33: Performing Preparation, Compilation, and Review Engagements

© 2021 Association of International Certified Professional Accountants. All rights reserved. 25

Compilation procedures work program

A compilation engagement program outlines the engagement approach and identifies other forms and

checklists to be used on the engagement. Different firms will tailor the nature and extent of

documentation required by the work program according to the uniqueness of their operations, their size,

and the complexity of their engagements. Although a work program is not required by the standards

governing compilation engagements, we recommend that a work program be completed for all

compilation engagements.

Each of the major steps in a compilation engagement should be included in the work program. Some of

these steps are completed in the planning phase by managers or partners. Others relate directly to the

staff’s obligations during the engagement. Note that the level of staffing may differ by firm and

engagement. The major steps include the following:

Make an acceptance or continuance decision. This step is usually completed by the partner or manager and documented on a client acceptance form or compilation planning checklist.

Consider whether the firm is independent. If the firm is not independent, the report on the compilation should be modified to disclose the lack of independence.

Document any step-down considerations. This step is usually completed by the partner or manager. If the engagement had initially been an audit or review, the partner or manager, or both, should consider whether it is appropriate to step down to a lower level of service and document his or her conclusion.

Establish an understanding with the client. Note that an engagement letter, signed by both management and the accountant, is required.

Determine scheduling and staff requirements. This step is usually completed by the partner or manager. Formal budgets are not required.

Obtain knowledge and understanding of the entity’s financial reporting framework. This step is typically documented on the client acceptance form, as previously discussed.

Consider the necessity of performing any other accounting services to enable compilation of financial statements.

Prepare or obtain an adjusted trial balance. The client may have prepared the adjusted trial balance or the accountant may prepare it. Typically, the preparation responsibility is outlined in the engagement letter. Reference any departures from the applicable financial reporting framework to the disclosure checklist.

Obtain revised or additional information, if necessary. Draft the financial statements (if engaged to prepare the financial statements) and the accountant’s

report. Be sure to reference trial balance accounts to combined amounts on the financial statements. Note any differences between combinations for the financial statements and those used for tax return presentation. Plan to use a disclosure checklist as subsequently discussed.

Add legends. Place a legend on each page of the financial statements, including the notes to the financial statements, with “See Accountant’s Compilation Report.”

Read the financial statements. Ensure that the financial statements are free from obvious error. Use the disclosure checklist subsequently discussed. Communicate any suspected fraud or illegal acts, unless they are clearly inconsequential.

Document consultation, resolve professional disputes, clear open items, and review notes. Review working papers. The in-charge accountant should document the review of the forms,

checklists, and working papers. Partner review and approval. This approval indicates that the financial statements are appropriate in

form and free from obvious material misstatement. The approval also indicates that the report form

Page 34: Performing Preparation, Compilation, and Review Engagements

© 2021 Association of International Certified Professional Accountants. All rights reserved. 26

is appropriate, and that the financial statements have been prepared (if the accountant is engaged to prepare the financial statements), approved, and processed in accordance with firm policies.

Bridging documents

Compilation engagement working papers may include documents that bridge the gap between the

client’s records and the compiled financial statements. These documents typically include a copy of the

working trial balance, any necessary adjustments, and the adjusted trial balance from which the financial

statements were prepared.

Disclosure checklist

Unless an explicit decision is made by management (not the accountant) to omit substantially all

disclosures from the compiled financial statements, all disclosures required by the applicable financial

reporting framework should be included with the financial statements. Because the list of possible

disclosures is very long, and the risk of inadvertently omitting a required disclosure can be high, we

recommend the use of a disclosure checklist for non-issuers.

Support for financial statement notes

In addition to ensuring that all necessary notes are included with the financial statements, working paper

documentation should include support for the notes to the financial statements. When all or substantially

all disclosures are omitted, the reasons for their omission should be documented.

Other items

We recommend that accountants also document any findings or issues that, in their judgment, are

significant (for example, the results of compilation procedures that indicate that the financial statements

could be materially misstated, including actions taken to address such findings, and to the extent the

accountant had any questions or concerns as a result of his or her compilation procedures or how those

issues were resolved). In addition, if circumstances indicate the presence of fraud or illegal acts, and the

accountant has communicated to the appropriate level of management or those charged with

governance, we suggest documenting those communications.

Page 35: Performing Preparation, Compilation, and Review Engagements

© 2021 Association of International Certified Professional Accountants. All rights reserved. 27

Knowledge check

3. Where is the documentation of the accountant’s knowledge and understanding of the entity’s financial reporting framework typically captured?

a. Bridging document. b. A compilation engagement program. c. A client acceptance form. d. A disclosure checklist.

4. Which statement is not correct?

a. Working paper documentation for a compilation should include support for the notes to the financial statements.

b. Working paper documentation for a compilation provides support for compliance with the standards that govern compilation engagements and the firm’s quality control procedures.

c. The extant working paper documentation for a compilation will vary depending on the circumstances of the engagement.

d. Working paper documentation for a compilation must include a signed, written engagement letter.

Page 36: Performing Preparation, Compilation, and Review Engagements

© 2021 Association of International Certified Professional Accountants. All rights reserved. 28

Summary We have just discussed the following points:

What constitutes a compilation engagement Entities for which compilations can be performed Differences between compilation and preparation engagements The performance requirements for a compilation engagement The minimum documentation requirements for a compilation engagement The forms, checklists, and other documentation in a compilation engagement

Page 37: Performing Preparation, Compilation, and Review Engagements

© 2021 Association of International Certified Professional Accountants. All rights reserved. 29

Practice questions 1. Identify the specialized industry information that should be included in the client information form for

the following industries:

a. Contractor

b. Not-for-profit organization

c. Retail business

2. Identify the specialized industry adjustments that will probably be made to the trial balance for the following industries:

a. Contractor

b. Not-for-profit organization

c. Retail business

Page 38: Performing Preparation, Compilation, and Review Engagements

© 2021 Association of International Certified Professional Accountants. All rights reserved. 30

3. Fill in the following chart that compares compilation, preparation, and audit engagements:

Compilation

engagement

Preparation

engagement

Audit

engagement

Level of assurance

Entities covered

Knowledge and understanding of the entity’s financial reporting framework

Knowledge of the client

Collect evidence

Applicable disclosures omitted

Accountant’s independence

Obtain an understanding of internal control

Engagement letter

Management representation letter

Case: Compliance with professional standards

You have been retained as an expert witness in an upcoming lawsuit involving the provision of accounting services by Jordan Kendall, CPAs, to Liberty Stores. Specifically, you have been asked to render an opinion about whether the engagement complied with professional standards applicable to a CPA firm in providing accounting and compilation services for the period from January 20X1December 20X3. The specifics of the case are as follows.

Description of the case

Liberty Stores, Inc. is a corporation that operates five large retail stores in Oregon. Mr. Thomas Sanders is the sole owner and president of Liberty Stores. Mr. Sanders employed Ms. Toni Blake as general manager of Liberty Stores from January 20X1December 20X3. In her capacity as general manager, Ms. Blake had the authority to write checks on multiple accounts for Liberty Stores, including all of the individual store accounts and one administrative account.

From January 20X1December 20X3, Mr. Sanders engaged Jordan Kendall to provide a compilation engagement for Liberty Stores. Each month, Jordan Kendall provided Mr. Sanders with the following: an accountant’s compilation report; a combined income statement; a combined balance sheet; income statements by retail location; a trial balance; supporting journal entries; and a disbursement register listing the check number, date, vendor, and amount of each check paid during the month. In addition, Jordan Kendall provided Mr. Sanders a payroll report at the end of each payroll period.

Page 39: Performing Preparation, Compilation, and Review Engagements

© 2021 Association of International Certified Professional Accountants. All rights reserved. 31

Case: Compliance with professional standards (continued)

At the request of Mr. Sanders, Jordan Kendall also provided monthly accounts payable services to Liberty Stores. In a letter dated January 20X1, Mr. Sanders instructed the individual store managers to approve all bills and forward them to Jordan Kendall for payment. Mr. Sanders also stated in that letter that “no checks are authorized to be drawn on the store accounts without Ms. Blake’s or my approval.” Accordingly, upon receipt of invoices from the various stores, Jordan Kendall prepared an accounts payable register which was forwarded to Mr. Sanders, who would then tell the accountants at Jordan Kendall which bills should be paid currently. In addition, Mr. Sanders authorized payments on some accounts by simply calling the accountants and instructing them to write a check.

Not all payments to vendors, however, were included in the accounts payable records. For example, some stores had access to their own checking accounts to pay out-of-pocket expenses, and Ms. Blake had the authority to write checks on the administrative account. Bills paid directly from these accounts by the store managers or Ms. Blake were not always forwarded to Jordan Kendall for inclusion in the accounts payable register. However, these payments were recorded in the disbursements journal provided to Mr. Sanders at the end of each month.

Mr. Sanders also engaged Jordan Kendall to provide monthly bank reconciliations for each of the accounts of Liberty Stores. Each month, bank statements with canceled checks and check stubs were received directly by or forwarded by employees of Liberty Stores to Jordan Kendall. Bank statements for the administrative account were sent directly to Ms. Blake, who faxed copies of the statements and the check stubs to Jordan Kendall. On several occasions, Kathi Johnson, an accountant for Jordan Kendall, verbally requested that Ms. Blake send the bank statements with canceled checks to Jordan Kendall. Likewise, on several occasions, Ms. Johnson informed Mr. Sanders that she was receiving copies of the bank statement for the administrative account without the canceled checks from Ms. Blake.

In March 20X4, Ms. Blake pled guilty to embezzlement involving Liberty Stores. In subsequent sentencing, Ms. Blake was ordered to make restitution of $250,000 worth of funds she fraudulently obtained from Liberty Stores. According to the declarations of Mr. Sanders and Ms. Brenda Hood (employee of Liberty Stores) in the criminal case against Ms. Blake, the following methods were used by Ms. Blake to embezzle from Liberty Stores:

1. Ms. Blake wrote checks on the administrative account to unauthorized parties, including herself, but recorded those checks in the check register sent to the accountants as written to authorized parties.

2. Ms. Blake wrote unauthorized checks on the administrative account for nonbusiness purposes.

In January 20X4, Mr. Sanders filed a civil complaint against Jordan Kendall alleging that the firm had been negligent in providing compilation engagements to Liberty Stores. Mr. Sanders specifically alleges that Jordan Kendall’s negligence resulted in the accounting firm failing to detect and report Ms. Blake’s embezzlement scheme.

Page 40: Performing Preparation, Compilation, and Review Engagements

© 2021 Association of International Certified Professional Accountants. All rights reserved. 32

Case: Compliance with professional standards (continued)

Required

After reviewing the complaint and discussing it with the attorney for Jordan Kendall, you have asked that all records pertaining to the Liberty Stores engagement be forwarded to your office. You have received these records in several large file boxes.

1. Define a compilation engagement. When are CPAs required to perform a compilation engagement and issue a compilation report?

2. Identify the performance requirements for compilation engagements. Why are the performance requirements for reviews and audits significantly more than those for a compilation?

3. What professional standards do you need to consider in reaching your conclusion about this case?

4. What records do you expect to find that will be of use to you in reaching your conclusion about whether the engagement was performed in accordance with professional standards?

5. What will you review for each item you identified in question 4?

Page 41: Performing Preparation, Compilation, and Review Engagements

© 2021 Association of International Certified Professional Accountants. All rights reserved. 33

Solutions

Performing Compilation Engagements

Knowledge check solutions

1.

a. Incorrect. A three-party relationship is an element of a compilation engagement.

b. Correct. A compilation engagement does not provide assurance and therefore is not one of the elements of a compilation engagement.

c. Incorrect. An applicable financial reporting framework is an element of a compilation engagement.

d. Incorrect. A written communication or report is an element of a compilation engagement.

2.

a. Correct. The accountant should consider the possible effect on the financial statements and communicate that consideration to management.

b. Incorrect. Qualified opinions can be issued for audit engagements, but not for compilations.

c. Incorrect. Issuing a disclaimer is not an option in a compilation engagement.

d. The accountant should only withdraw from the engagement when management fails to provide additional evidence or make appropriate revisions.

3.

a. Incorrect. A bridging document bridges the gap between a client’s records and compiled financial statements.

b. Incorrect. This identifies the accountant’s engagement approach.

c. Correct. This is filed in the permanent file or latest working paper file.

d. Incorrect. This ensures that the financial statement disclosures are complete.

Page 42: Performing Preparation, Compilation, and Review Engagements

© 2021 Association of International Certified Professional Accountants. All rights reserved. 34

4.

a. Correct. This statement is not correct. Support for the notes to the financial statements is not required documentation of a compilation engagement.

b. Incorrect. This statement is correct. Documentation provides evidence of compliance with applicable standards for conducting a compilation engagement.

c. Incorrect. This statement is correct. The amount of working paper documentation will vary by engagement and circumstance.

d. Incorrect. This statement is correct. Compilation engagements must include a signed, written engagement letter.

Practice question solutions

1.

a. Types of projects normally constructed by the company, including locations of projects, normal length of construction period, financing projects, general bonding requirements, use of subcontractors or in-house labor, and so on.

The types of contracts into which the company normally enters and the company’s normal proposal and negotiation process.

Revenue recognition methods for both book and tax purposes.

Ability to provide accurate estimates of costs to complete contracts in progress.

Joint venture or similar arrangements between the company and other parties.

Specialized accounting records: contract billings, contract costs, subcontractors, contract status reports.

b. Types of program services.

Revenue services.

Need for compliance audit.

Use of fund accounting.

Responsibilities of the board of directors.

c. Inventory valuation method.

Product maintenance contracts.

Revenue recognition method.

Page 43: Performing Preparation, Compilation, and Review Engagements

© 2021 Association of International Certified Professional Accountants. All rights reserved. 35

2.

a. Receivables: Separate contract retentions receivable from other contract receivables.

Inventories: Determine that all unused inventory returned from jobs has been recorded at the lower of cost or market.

Other assets: Determine that workers’ compensation, general liability, and excess coverage (umbrella) insurance premiums have been properly accrued.

Liabilities: Segregate subcontractor retentions payable.

Construction contracts: Reconcile billings by job to the subsidiary contract records.

Reconcile prior period revenue to the prior year’s financial statements or tax return.

Reconcile job costs by job to the subsidiary contract records.

Determine net overbilled and unbilled amounts for contracts in progress.

b. Receivables: Determine appropriate valuation of pledges.

Other assets: Determine appropriate valuation of donated assets, including collections.

Revenues: Determine appropriate valuation of donated services.

c. Inventories: Determine that inventories have been appropriately valued.

Page 44: Performing Preparation, Compilation, and Review Engagements

© 2021 Association of International Certified Professional Accountants. All rights reserved. 36

3.

Compilation engagement

Preparation engagement

Audit engagement

Level of assurance None None Reasonable

Entities covered Nonpublic (Non-issuers)

Nonpublic (Non-issuers) Issuers and non-issuers

Knowledge and understanding of the entity’s financial reporting framework

Knowledge and understanding of the entity’s financial reporting framework sufficient to ensure the financial statements are in appropriate form

Knowledge and understanding of the entity’s financial reporting framework sufficient to enable preparation of financial statements

Extensive knowledge of the entity’s financial reporting framework, economy, and the relevant industry

Knowledge of the client Understanding of the client’s business sufficient to ensure the financial statements are in appropriate form.

Understanding of the client’s business sufficient to enable preparation of the financial statements

Extensive understanding of the client’s business required

Collect evidence Not required Not required Extensively required

Applicable disclosures omitted

Substantially all disclosures may be omitted

Substantially all disclosures may be omitted

Inadequate disclosures require qualified or adverse opinion

Accountant’s independence Must be considered; lack of independence should be disclosed in a final paragraph to accountant’s compilation report

Not required to consider Must be considered; lack of independence results in disclaimer of opinion

Obtain an understanding of internal control

Not required Not required Extensively required for non-issuers or issuers; separate report on internal control issued for public companies

Engagement letter Required Required Required

Management representation letter

Not prohibited or required

Not required Required

Page 45: Performing Preparation, Compilation, and Review Engagements

© 2021 Association of International Certified Professional Accountants. All rights reserved. 37

Case: Compliance with professional standards solutions

A compilation engagement involves applying accounting and financial reporting expertise to 1.assist management in the presentation of financial statements and reporting on those financial statements in accordance with SSARSs. A compilation is a significantly lower level of service than either an audit or review engagement, and provides no assurance on the financial statements being presented.

CPAs are required to perform a compilation engagement in accordance with SSARSs and issue a compilation report when engaged to do so by the client.

The performance requirements for compilations are set forth in SSARSs and include the 2.following:

a. Establish an understanding with the client. This understanding with the client should be written, in the form of an engagement letter or other suitable written contract, and signed by both management and the accountant.

b. Have or obtain knowledge and understanding of the entity’s financial reporting framework.

c. Read the financial statements to ensure that they are free from obvious errors.

d. Obtain additional information if the CPA becomes aware that the information supplied is incorrect, incomplete, or otherwise unsatisfactory.

The performance requirements for compilation, review, and audit are significantly different due to the different levels of assurance provided by each service. Compilations have the fewest performance requirements, given that the service provides no assurance on financial statements.

The pertinent professional standards to consider in this case include the following: 3.

a. The AICPA Code of Professional Conduct.

b. The Oregon Board of Public Accountancy Rules for Certified Public Accountants.

c. AICPA SSARS, specifically AR-C section 60 and AR-C section 80.

d. AICPA Statements on Quality Control Standards (SQCSs).

You should expect to find the following records that will be of use to you in reaching your 4.conclusion about whether the engagement was performed in accordance with professional standards:

a. Engagement letters documenting the understanding reached with Liberty Stores regarding the nature and scope of services to be performed by Jordan Kendall.

b. Accountant’s reports.

c. Compiled financial statements.

d. Supporting documentation including trial balances, journal entries, and so on.

e. Bank reconciliations.

f. Peer review letters, if Jordan Kendall was reviewed during this period.

Page 46: Performing Preparation, Compilation, and Review Engagements

© 2021 Association of International Certified Professional Accountants. All rights reserved. 38

You will be reviewing the preceding items for the following: 5.

a. Engagement letter. Evidence that Jordan Kendall and Liberty Stores had reached an understanding regarding the nature and scope of the compilation services performed. In particular, the engagement letter should be reviewed for any discussion of the accountant’s responsibility for detecting or disclosing errors, irregularities, or illegal acts.

b. Accountant’s report. Evidence that the report is appropriate in form for the engagement, and supported by the working paper documentation. Certainly, any exceptions noted in the report should be considered.

c. Compiled financial statements. Evidence that the financial statements appear to be “appropriate in form” for the client and the industry in which it operates and “free from obvious material errors,” including organizational, mathematical, and presentation and disclosure errors.

d. Supporting documentation. Evidence that Jordan Kendall had an adequate understanding of the entity’s financial reporting framework and complied with professional standards in conducting the engagement.

e. Bank reconciliations. Evidence that the bank reconciliations were appropriately completed, reconciling the general ledger cash balances to bank statement balances. (Note that comparisons of payee per the check, payee per the check stub, and endorsee are not part of the typical reconciliation process. Although these comparisons might be used during an audit of financial statements, they are not required for preparation or compilation engagements.)

f. Peer review letters. If available for this time period, undergoing peer review is evidence of a firm’s ongoing quality assurance process. Further, an unqualified peer review letter provides additional evidence of having complied with professional standards in providing accounting services during the time under question.

Page 47: Performing Preparation, Compilation, and Review Engagements

© 2021 Association of International Certified Professional Accountants. All rights reserved. 39

Performing Review Engagements

Learning objectives

Identify the general principles and objective of review engagements.

Recognize the elements that make up review engagements.

Identify the performance standards for a review engagement.

Recognize the documentation requirements for a review engagement.

Recognize the effects of a change in level of service from an audit to a review.

Page 48: Performing Preparation, Compilation, and Review Engagements

© 2021 Association of International Certified Professional Accountants. All rights reserved. 40

Review general principles and objective This section provides an overview of the objective and elements of review engagements. It includes a

discussion of materiality considerations.

Objective

The objective of the accountant when performing a review of financial statements is to obtain limited

assurance as a basis for reporting whether the accountant is aware of any material modifications that

should be made to the financial statements for them to be in accordance with the applicable financial

reporting framework, primarily through the performance of inquiry and analytical procedures. In order to

conclude that he or she has obtained limited assurance, the accountant is required to accumulate review

evidence through the application of procedures, including inquiry and analytical procedures, supported by

a management representation letter. The accountant is required to evaluate whether sufficient

appropriate review evidence has been obtained from the procedures performed and, if not, the

accountant is required to perform other procedures judged by the accountant to be necessary in the

circumstances in order to form a conclusion on the financial statements.

Review engagements are assurance services that may be provided only to nonpublic entities, referred to

in the literature as non-issuers. A non-issuer is any entity that does not issue securities under Section 12

of the Securities Exchange Act of 1934 and is therefore not subject to the Sarbanes-Oxley Act of 2002, or

the rules of the SEC.

Comparison to audit engagements

While there are certainly differences between review and audit engagements, these engagements also

have similarities, including that they both

are attest engagements. require the accountant to be independent. require the accountant to issue a written report. provide assurance on the financial statements.

Though they do have some similarities, the engagements differ considerably. The following paragraphs

provide a brief overview of their differences.

The audit engagement is the highest level of attest service accountants can provide to their clients with

respect to financial statements, requiring more extensive procedures and providing a higher level of

assurance than a review engagement. The purpose of an audit is to enhance the degree of confidence

that intended users can place in the financial statements by providing an auditor’s opinion on whether the

financial statements are presented fairly, in all material respects, in accordance with the applicable

financial reporting framework. An audit conducted in accordance with generally accepted auditing

Page 49: Performing Preparation, Compilation, and Review Engagements

© 2021 Association of International Certified Professional Accountants. All rights reserved. 41

standards and relevant ethical requirements enables the auditor to form that opinion. An audit involves

performing procedures to obtain audit evidence about the amounts and disclosures in the financial

statements. The procedures selected depend on the auditor’s judgment, including the assessment of the

risks of material misstatement, whether due to fraud or error. In making those risk assessments, the

auditor considers internal controls relevant to the entity’s preparation and fair presentation of the

financial statements to design audit procedures that are appropriate in the circumstances, but not for

expressing an opinion on the effectiveness of internal control. Simply put, for an audit to provide a higher

level of assurance (reasonable assurance), the audit must be conducted with a higher level of rigor than a

review of financial statements.

In contrast, the review engagement provides limited assurance on the financial statements. Limited

assurance is defined as assurance that is less than the reasonable assurance obtained in an audit

engagement, but is at an acceptable level as the basis for the conclusion in the accountant’s review

report. As such, reviews are less in scope than audit engagements and require fewer procedures to

obtain a lower level of assurance. The review specifically does not contemplate obtaining an

understanding of the entity’s internal control, assessing fraud risk, or obtaining enough appropriate audit

evidence. Accordingly, in a review, the accountant does not obtain assurance that he or she will become

aware of all significant matters that would be disclosed in an audit.

Elements of a review engagement

Any review engagement involves five elements:

A three-party relationship involving management, an accountant, and intended users An applicable financial reporting framework Financial statements or financial information Sufficient appropriate review evidence Materiality A written communication or report

Three-party relationship

The three-party relationship includes management or a responsible party, an accountant in the practice

of public accounting (as defined by the code), and intended users of the financial statements or financial

information. Although it is often the case that management and the intended users may be the same,

every so often intended users may be from different entities, like a bank or potential investor.

In this three-party relationship, the accountant is required to obtain management’s agreement that it

acknowledges and understands its responsibilities, including selecting the financial reporting framework,

designing and implementing internal controls, preventing and detecting fraud, etc. That these are

management responsibilities does not preclude the accountant from making suggestions about the

financial statements or even preparing them, in whole or in part, based on information provided by

management. If, however, management is unwilling to accept its responsibilities, the accountant is

precluded from performing the review engagement.

Page 50: Performing Preparation, Compilation, and Review Engagements

© 2021 Association of International Certified Professional Accountants. All rights reserved. 42

To perform a review engagement, accountants are required to possess or obtain an understanding of the

industry in which the entity operates, including the accounting principles and practices generally used in

the industry, sufficient to enable them to review financial statements that are appropriate for the entity

operating in that industry.

Intended users of the financial statements are the person or persons or class of persons who understand

the limitations of the review engagement and the financial statements. Although the intended users of

the financial statements are an integral part of the three-party relationships found in any review

engagement, the accountant has no obligation to identify the intended users.

On occasion, intended users, such as bankers or regulators, may require or ask the client to ask that

additional procedures be performed in conjunction with the review engagement. For example, an

intended user may ask that certain agreed-upon procedures such as a review of account reconciliations

be performed. Accountants may perform these additional services in conjunction with the review

engagement if the accountant adheres to professional standards regarding those services.

Applicable financial reporting framework

A financial reporting framework is a set of criteria used to determine measurement, recognition,

presentation, and disclosure of all material items that appear in the financial statements (for example,

generally accepted accounting principles [GAAP], International Financial Reporting Standards

promulgated by the International Accounting Standards Board, or a special purpose framework).

Management, or those charged with governance, is ultimately responsible for selecting the entity’s

applicable financial reporting framework as well as choosing among individual accounting policies when

the framework provides acceptable alternatives.

Financial statement or financial information

Accountants may be engaged to review complete sets of financial statements or individual financial

statements. The requirements of the applicable financial reporting framework determine what

constitutes a complete set of financial statements.

In addition, review engagements can be undertaken for other historical financial information including

specified elements, accounts or items of a financial statement, supplementary or required supplementary

information, and financial information contained in a tax return. Accountants may not undertake a review

of pro forma financial information. Reviews of pro forma financial information are to be performed in

accordance with Statements on Standards for Attestation Engagements.

Evidence

The accountant is required to design and perform analytical procedures, make inquiries, and perform

other procedures, as appropriate, to obtain limited assurance as a basis for reporting whether the

accountant is aware of any material modifications that should be made to the financial statements in

order for the statements to be in accordance with the applicable financial reporting framework based on

the accountant’s

Page 51: Performing Preparation, Compilation, and Review Engagements

© 2021 Association of International Certified Professional Accountants. All rights reserved. 43

a. understanding of the industry, b. knowledge of the entity, and c. awareness of the risk that the accountant may unknowingly fail to modify the accountant's review

report on financial statements that are materially misstated.

When performing a review engagement, the accountant is required to evaluate whether sufficient

appropriate review evidence has been obtained from the procedures performed and, if not, the

accountant is required to perform other procedures judged by the accountant to be necessary in the

circumstances to be able to form a conclusion on the financial statements.

The nature, timing, and extent of procedures for gathering review evidence are limited relative to

an audit.

Materiality

The concept of materiality is embedded throughout review engagements, including (1) a conclusion that

notes the accountant is not aware of any material modifications necessary for the financial statements,

(2) the design and performance of inquiry and analytical procedures that reflects a focus on material

items, and (3) the evaluation of uncorrected misstatements and inadequate disclosures in which the

accountant considers their effects both individually and in the aggregate. In addition, Statements on

Standards for Accounting and Review Services (SSARSs) explicitly requires the accountant to determine

materiality for the financial statements as a whole, including disclosures, and apply this materiality in

designing and evaluating results obtained from those procedures.

An accountant’s consideration of materiality is made in the context of the applicable financial reporting

framework. Some financial reporting frameworks discuss the concept of materiality in the context of the

preparation and presentation of financial statements, where others are silent on the matter. Although

financial reporting frameworks may discuss materiality in different terms, they generally share the

following elements:

Reasonable user test. Misstatements, including omissions, are considered to be material if there is a substantial likelihood that, individually or in the aggregate, they would influence the judgment made by a reasonable user based on the financil statements.

Contextually driven: Judgments about materiality are made in light of surrounding circumstances and affected by the size or nature of a misstatement or a combination of both.

Number and nature. Judgments about materiality involve both qualitative and quantitative considerations. Qualititative factors are those that are material due to their nature, but the dollar amount alone would be considered immaterial. For example, a small misstatement that converted a net loss to net income could be considered qualitatively material.

Collective needs. Judgements about matters that are material to users of the financial statements are based on a consideration of the common financial information needs of users as a gourp. The possible effect of misstatements on specific individual users, whose needs may vary widely, is not considered.

In those circumstances where a financial reporting framework does not include a discussion of

materiality, accountants should consider these characteristics as a frame of reference.

Page 52: Performing Preparation, Compilation, and Review Engagements

© 2021 Association of International Certified Professional Accountants. All rights reserved. 44

Ultimately, the accountant’s determination of materiality is a matter of professional judgment and is

affected by the accountant’s perception of the needs of the intended users of the financial statements. In

this context, it is reasonable for accountants to assume that users

have a reasonable knowledge of business and economic activities and accounting and a willingness to study the information in the financial statements with reasonable diligence;

understand that financial statements are prepared, presented, and reviewed to levels of materiality;, recognize the uncertainties inherent in the measurement of amounts based on the use of estimates,

judgment, and the consideration of future events; and make reasonable economic decisions on the basis of the information in the financial statements.

Although materiality should be determined and applied before designing procedures and evaluating

results, the accountant may become aware of information during the review that would have affected the

materiality determination had it been known at the time. The accountant may become aware of this

information due to a change in circumstances for the entity that occurs during the review (e.g. the

deicison to dispose of a major segment), or as a result of performing review procedures (e.g. actual

financial results are going to be signficantly different than the anticipated results used to determine

materiality). In these circumstances, accountants should revise materiality for the financial statements

taken as a whole.

A written report

Accountants performing review engagements are required to provide a written review report unless the

accountant withdraws from the engagement.

Knowledge check

1. In a review engagement, the accountant obtains ___________ assurance as a basis for reporting whether the accountant is aware of any material modifications that should be made to the financial statements for them to be in accordance with the applicable financial reporting framework.

a. Reasonable. b. Limited. c. No. d. Absolute.

Page 53: Performing Preparation, Compilation, and Review Engagements

© 2021 Association of International Certified Professional Accountants. All rights reserved. 45

Review performance standards The following are major performance standards for a review engagement and are discussed in further

detail in subsequent sections:

Establish an understanding with the client Communicate with management or those charged with governance Have appropriate knowledge of the industry Have appropriate knowledge of the client Obtain sufficient appropriate review evidence Reconcile financial statements to underlying records Perform inquiry and analytical procedures Obtain additional information if the information supplied is incorrect, incomplete, or otherwise

unsatisfactory Obtain a management representation letter Read the financial statements

Generally speaking, performance standards for any engagement are designed to enable the accountant

to accumulate review evidence that will provide a reasonable basis for obtaining limited assurance that

there are no material modifications that should be made to the financial statements in order for the

statements to be in conformity with the applicable financial reporting framework. The accountant should

apply professional judgment in determining the specific nature, timing, and extent of review procedures,

with procedures tailored to the engagement based on the accountant’s knowledge of the industry and

entity. Although review evidence obtained through analytical procedures, inquiry, and a management

representation letter is typically sufficient to provide the accountant with a reasonable basis for obtaining

limited assurance, the accountant’s professional judgment should still apply. If, for example, the

accountant considers additional procedures necessary to obtain limited assurance on a particular

engagement, the accountant should perform those procedures.

Note that accountants are required to comply with the performance standards discussed in the following

sections whenever they have been engaged to review financial statements. One exception is provided for

reviews of interim financial information when the following requirements are met:

The entity’s latest annual financial statements have been audited by the accountant or a predecessor accountant.

The accountant either — has been engaged to audit the entity’s current year financial statements, or — has audited the entity’s latest annual financial statements and, when it is expected that the

current year financial statements will be audited, the appointment of another accountant to audit the current year financial statements is not effective prior to the beginning of the period covered by the review.

The client entity prepares its interim financial information in accordance with the same financial reporting framework that was used to prepare the annual financial statements.

Page 54: Performing Preparation, Compilation, and Review Engagements

© 2021 Association of International Certified Professional Accountants. All rights reserved. 46

If these conditions are met, accountants should conduct the review in accordance with auditing standards,

particularly AU-C section 930, Interim Financial Information,1 rather than the performance standards set

forth by SSARSs. Although both reviews performed in accordance with SSARSs and those performed in

accordance with auditing standards are limited assurance engagements, there are some significant

differences. For example, those engagements performed in accordance with the auditing standards require

accountants to update their knowledge of the entity’s internal control, and the issuance of a review report is

optional. For reviews conducted in accordance with SSARSs, knowledge of the entity’s internal control is

not required, and a review report must be issued.

Establish an understanding

Accountants performing review engagements are required to comply with AR-C section 60, General

Principles for Engagements Performed in Accordance with Statements on Standards for Accounting and

Review Services. As such, accountants must follow the guidance in AR-C section 60 regarding the pre-

engagement decisions to accept or continue a client. In addition, once the decision is made to accept or

continue the client, accountants performing review engagements must also follow the guidance

regarding a written, signed agreement between the client and accountant. The written agreement is

typically in the form of an engagement letter, but a formal contract is also suitable. An oral understanding

of the terms of the engagement is not enough to meet the requirement. Although the performance

standards for an engagement to review financial statements require the accountant to have knowledge

of the industry and client in order to perform the engagement, it is advisable to obtain some level of

understanding prior to determining the scope of the engagement.

Communicate with management or those charged with governance

Accountants performing review engagements should be communicating with management or those

charged with governance as necessary and on a timely basis throughout the engagement. The

significance and nature of the matter determines the need and timing of the communication. After

executing the engagement letter, communications with management or those charged with governance

might involve significant findings, matters that may require report modification, or significant difficulties

encountered during the review.

One of the more difficult situations involving communication between the accountant and client arises

when accountants encounter situations that indicate fraud or an illegal act. When an accountant

suspects that fraud or an illegal act may have occurred, the accountant should communicate the matter,

unless it is clearly inconsequential, to an appropriate level of management. If the suspected fraud or

illegal act involves senior management, the accountant should communicate the matter to an individual

or group at the highest level within the entity, such as the manager (owner) or board of directors. If the

1 All AU-C and AR-C sections can be found in AICPA Professional Standards.

Page 55: Performing Preparation, Compilation, and Review Engagements

© 2021 Association of International Certified Professional Accountants. All rights reserved. 47

suspected fraud or illegal act involves an owner of the business, the accountant should consider

withdrawing from the engagement.

The disclosure of possible fraud or illegal acts to parties other than the client’s senior management

ordinarily is not part of the accountant’s responsibility and ordinarily would be precluded by ethical or

legal obligations of confidentiality. Because potential conflicts between ethical and legal obligations for

confidentiality may be complex, the accountant may wish to consult legal counsel before discussing

such matters with parties other than the client.

Have or obtain knowledge of the industry

Accountants are required to have knowledge of the accounting principles and practices of the industry

and the method of applying those accounting principles and practices. The purpose of industry

knowledge in a review engagement is to assist the accountant with determining the specific nature,

timing, and extent of review procedures to be performed. Specifically, knowledge of the industry is

ordinarily used by the accountant in constructing inquiries and analytical procedures adequate to provide

a reasonable basis for expressing limited assurance.

Accountants are not required to have appropriate knowledge of the industry at the time the engagement

is accepted. The engagement can be accepted with the understanding that the accountant will obtain the

appropriate industry knowledge. The accountant may obtain this knowledge by consulting AICPA guides,

industry publications, financial statements of other entities in this industry, textbooks and periodicals,

appropriate continuing professional education, or individuals knowledgeable about the industry.

Although SSARSs do not specify when that knowledge must be obtained (before the engagement

commences or simply before the engagement is completed), the complexity of the industry should

determine this timing. For example, consider two industriesone relatively complex, the other not. For an

accountant new to the relatively complex industry, obtaining an understanding of the industry earlier may

be important in establishing an understanding with the client and executing an appropriate engagement

letter. Conversely, for an accountant new to the relatively less complex industry, that understanding may

be obtained as needed throughout the engagement.

Have or obtain knowledge of the client

Accountants are required to have knowledge of the client’s business and an understanding of the

accounting principles and practices used by the client sufficient to enable them to review the financial

statements. Like knowledge of the industry, the accountant does not have to have this knowledge of the

client at the time the engagement is accepted, if that knowledge is obtained before the engagement is

completed.

Page 56: Performing Preparation, Compilation, and Review Engagements

© 2021 Association of International Certified Professional Accountants. All rights reserved. 48

Most often, knowledge of the client is gained through prior experience with the client, inquiries of

personnel, observation of operations and review of previous financial statements. Specifically, the

accountant is required to have knowledge of the following:

The entity’s organization and business The accounting principles and practices used by the entity The nature of the entity’s assets, liabilities, revenues, and expenses

The accountant will have general knowledge of the entity’s production, distribution, and compensation

methods, types of products and services, operating locations, and material transactions with related

parties.

In obtaining an understanding of the client’s accounting policies and practices, the accountant should be

alert to unusual accounting policies and procedures that come to the accountant’s attention because of

knowledge of the industry.

Obtain sufficient appropriate review evidence

To support a conclusion that provides limited assurance on the financial statements, accountants

conducting a review engagement should obtain sufficient appropriate review evidence. Typically that

evidence can be obtained from (1) reconciling the financial statement to underlying records, (2)

performing inquiry and analytical procedures, (3) evaluating evidence obtained from other procedures

performed, and (4) obtaining a written representation letter from management. Ultimately, the

accountant will evaluate whether the combined evidence obtained from these procedures is sufficient

and appropriate. If not, the accountant should perform additional procedures to be able to support a

conclusion. If sufficient appropriate review evidence cannot be obtained, an accountant should consider

his or her ability to complete the engagement. Note that this situation may arise even though the

accountant has not become aware of a matter(s) that causes him or her to believe the financial

statements may be materially misstated.

Reconciliation of the financial statements

In any review engagement, the accountant should obtain evidence that the financial statements agree or

reconcile with the underlying accounting records. This evidence can be obtained by comparing the

financial statements with the accounting records, such as the general ledger, a consolidating schedule

derived from those records, or other supporting data in the entity’s records.

Inquiry and analytical procedures

Performing inquiries and analytical procedures are the principal means of obtaining a reasonable basis

for expressing limited assurance. The design of inquiry and analytical procedures should be broadly

based on the accountant’s understanding of the industry and knowledge of the client. In addition, when

designing these procedures, the accountant should be aware of the risk that the accountant may

unknowingly fail to modify the review report on financial statements that are materially misstated.

Page 57: Performing Preparation, Compilation, and Review Engagements

© 2021 Association of International Certified Professional Accountants. All rights reserved. 49

Although analytical procedures and inquiry are required for review engagements, professional judgment

determines the extent to which those procedures or others should be performed. In addition, the

Accounting and Review Services Committee has noted that as an assurance engagement, the review

engagement should not be considered simply an exercise in performing analytical procedures and

making management inquiries. Instead, the accountant should concentrate on accumulating sufficient

review evidence to provide limited assurance that there are no material modifications that should be

made to the financial statements. In making this determination, accountants should exercise their

judgment and focus on those areas where the accountant believes there are increased risks of

misstatements, also considering the following:

The nature and materiality of the items The likelihood of misstatement Knowledge obtained during current and previous engagements The stated qualifications of the entity’s accounting personnel The extent to which a particular item is affected by management’s judgment Inadequacies in the entity’s underlying financial data

Note that results of the accountant’s analytical procedures and inquiries may modify the accountant’s

risk awareness. For example, the response to an inquiry that cash has not been reconciled for several

months may revise the accountant’s awareness of risks relative to the cash account.

Evidence obtained from other procedures

The standards for review engagements encourage accountants to remain alert to information that may

indicate related parties or related party transactions that have not previously been identified. In addition,

for transactions outside the normal course of business, accountants should inquire about the nature of

those transactions and the involvement of any related parties.

Throughout the conduct of the review engagement, the accountant should accumulate misstatements,

including inadequate disclosures. These accumulated misstatements should be evaluated both

individually and in the aggregate to determine whether material modification should be made to the

financial statements for them to be in accordance with the applicable financial reporting framework. In

evaluating these uncorrected misstatements, the accountant should consider the following:

The nature, cause (if known), and amount of misstatements Whether the misstatements originated in the preceding year The potential effect of the misstatement on future periods The appropriateness of offsetting a misstatement of an estimated amount with a misstatement of an

item capable of precise measurement Recognition that an accumulation of immaterial misstatements in the balance sheet could contribute

to material misstatements in the future

If during the performance of review procedures the accountant becomes aware that the information

supplied is incorrect, incomplete, or otherwise unsatisfactory, the accountant may need to obtain

additional information. In these circumstances, the accountant should request that management

consider the effect of these matters on the financial statements and communicate the results of its

consideration to the accountant. Subsequently, if the accountant believes that the financial statements

Page 58: Performing Preparation, Compilation, and Review Engagements

© 2021 Association of International Certified Professional Accountants. All rights reserved. 50

may be materially misstated, the accountant should perform additional procedures deemed necessary to

obtain limited assurance on the financial statements. This judgment is based on results of inquiries or

procedures employed to comply with the standards for review, knowledge gained from prior

engagements, the financial statements, and the adjusted trial balance.

If significant components are reviewed or audited by other accountants, the accountant is required to

obtain reports from the other accountants as a basis, in part, for the review report. The accountant may

request a written confirmation of independence from the other accountant(s).

In obtaining additional or revised information, the accountant may perform procedures not ordinarily

required for a review engagement. For example, the accountant may review an account reconciliation or

confirm a cash balance to obtain additional or revised information. Although these procedures exceed

the performance requirements of the review, they do not change the nature of the engagement to a

higher level of assurance. These procedures should be documented in the working papers, but no

reference is made to them in the review report.

Ultimately, if the accountant concludes that the financial statements are materially misstated, the review

report will need to be modified to reflect a departure from the applicable financial reporting framework. A

full discussion of reporting issues is beyond the scope of this course.

Management representation letter

Written representations (management representation letters) are an important source of review

evidence. Accountants should obtain a management representation letter for all review engagements.

The management representation letter confirms oral representations made during the planning and

performance of the review engagement, and should be dated no earlier than the date of the accountant’s

report. An illustrative management representation letter is included in AR-C section 90, Review of

Financial Statements.

Failure to obtain the representation letter represents a scope limitation that precludes the accountant

from issuing a review report. In addition, the client’s refusal to supply a representation letter normally

precludes the accountant from stepping down to a compilation service and issuing a compilation report.

The purpose of the representation letter is to require management or owners to acknowledge their

responsibility for the financial statements. Even when the accountant has previously compiled the financial

statements that are under review, management should acknowledge its primary responsibility for those

financial statements. In addition, like an engagement letter, the representation letter clarifies roles and

responsibilities and may serve to reduce misunderstandings between the accountant and client.

The management representation letter should be addressed to the accountant and should cover all

financial statements and all periods covered by the accountant’s review report. The representation letter

should be signed by those members of management whom the accountant believes are responsible for

and knowledgeable about the matters covered in the representation letter. Ordinarily, the chief executive

officer and chief financial officer or others with equivalent positions in the entity, should sign the

representation letter. Occasionally this requirement poses problems when the financial statements are

Page 59: Performing Preparation, Compilation, and Review Engagements

© 2021 Association of International Certified Professional Accountants. All rights reserved. 51

comparative, and management has changed during the time being presented. The first paragraph of the

representation letter ordinarily allows management to limit its responses to his or her best knowledge

and belief. Accordingly, even if management has changed over the period covered by the review

engagement, current management should still provide this representation letter for a review report to be

issued.

Specific written representations obtained by the accountant will depend on the circumstances of the

engagement and the nature and basis of presentation of the financial statements. At a minimum, the

representation letter, addressed to the accountant, will include the following:

Management has fulfilled its responsibility for the preparation and fair presentation of the financial statements, in accordance with the applicable financial reporting framework, as set out in the terms of the engagement.

Management acknowledges its responsibility for designing, implementing, and maintaining internal control relevant to the preparation and fair presentation of financial statements, including its responsibility to prevent and detect fraud.

Management has provided the accountant with all relevant information and access, as agreed upon in the terms of the engagement.

Management has responded fully and truthfully to all of the accountant’s inquiries. All transactions have been recorded and are reflected in the financial statements. Management has disclosed to the accountant its knowledge of any allegations of fraud or suspected

fraud affecting the entity involving management, employees who have significant roles in internal control, or others when the fraud could have a material effect on the financial statements.

Management has disclosed to the accountant its knowledge of any allegations of fraud or suspected fraud affecting the entity’s financial statements communicated by employees, former employees, regulators, or others.

Management has disclosed to the accountant all known instances of noncompliance or suspected noncompliance with laws and regulations whose effects should be considered when preparing financial statements.

A summary should be included in, or attached to, the written representation stating whether management believes that the effects of uncorrected misstatements are immaterial, individually or in the aggregate, to the financial statements as a whole.

Management has disclosed to the accountant all known or actual or possible litigation and claims whose effects should be considered when preparing the financial statements, and it has appropriately accounted for and disclosed such litigation and claims in accordance with the applicable financial reporting framework.

A statement should be included indicating whether management believes significant assumptions used by it in making accounting estimates are reasonable.

Management has disclosed to the accountant the identity of the entity’s related parties and all of the related party relationships and transactions of which it is aware and it has appropriately accounted for and disclosed such relationships and transactions.

All events occurring subsequent to the date of the financial statements and for which the applicable financial reporting framework requires adjustment or disclosure have been adjusted or disclosed.

The representation letter ordinarily should be tailored to include additional appropriate representations from

management relating to matters specific to the entity’s business or industry. For example, in the

construction industry, a company that uses the percentage-of-completion method to recognize revenue

Page 60: Performing Preparation, Compilation, and Review Engagements

© 2021 Association of International Certified Professional Accountants. All rights reserved. 52

may represent that they have enough means to adequately assess the percentage of completion on

contracts.

On occasion, it may be necessary to update the management representation letter. An update may be

required when the original letter is dated as of the completion of procedures, but the report is issued

much later, or when a material subsequent event occurs after the date of the original representation

letter, but before the report is issued. Updated management representation letters may also be necessary

when a predecessor accountant is asked to reissue a report on financial statements of a prior period to

present those financial statements comparatively with subsequent financial statements.

The updated management representation letter should state whether any information has come to

management’s attention that would cause management to believe that any of the previous

representations should be modified. The updated representation letter should also state whether any

events have occurred subsequent to the balance sheet date of the latest financial statements reported

on by the accountant that would require adjustment to or disclosure in the financial statements.

Because the accountant is concerned with events occurring through the date of the report that may

require adjustment to or disclosures in the financial statements, the management representation letter

should be dated no earlier than the date of the accountant’s report. The accountant does not need to

have physical control of the management representation letter as of the review report date, provided that

management has acknowledged that they will sign the representation letter without modification and it is

received prior to release of the review report.

Professional skepticism

Accountants performing review engagements are required to plan and perform the review with

professional skepticism, recognizing that circumstances may exist that cause the financial statements to

be materially misstated. Professional skepticism is necessary to the critical assessment of review

evidence, and is exercised by neither assuming that management is dishonest or unquestionably honest.

The exercise of professional skepticism includes being alert to items like review evidence that

contradicts other review evidence obtained, or information that brings into question the reliability of

responses to inquiries and other information or records to be used as review evidence.

Read the financial statements

The accountant is required to read and ascertain whether the financial statements are free from obvious

errors. The reading of the financial statements should be performed toward the end of the engagement

when the accountant is virtually ready to present the reviewed financial statements to the client. The

accountant should carefully scrutinize the financial statements for mathematical or clerical mistakes and

mistakes in the application of accounting principles, including inadequate disclosure.

Page 61: Performing Preparation, Compilation, and Review Engagements

© 2021 Association of International Certified Professional Accountants. All rights reserved. 53

Knowledge check

2. Accountants performing review engagements

a. Are not obliged to search for fraud or illegal acts. b. Are obliged to search for fraud or illegal acts. c. Are obliged to have or obtain expert knowledge of the industry. d. Are not obliged to communicate evidence of fraud or illegal acts to management or those

charged with governance.

3. In performing a review engagement, an accountant

a. Should never perform additional procedures beyond those required for a review. b. May tailor the procedures performed to the client’s industry and business. c. Must have a signed engagement letter, but is not required to obtain a management

representation letter. d. Must understand and test internal controls.

Page 62: Performing Preparation, Compilation, and Review Engagements

© 2021 Association of International Certified Professional Accountants. All rights reserved. 54

Documentation for review engagements The goal of documentation is to enable an experienced accountant who has no previous connection to

the review to understand the nature, timing, and extent of procedures, the results of the procedures

performed and evidence obtained and the significant findings, conclusions reached, and judgments

made in the engagement. Documentation is necessary to support compliance with the standards

governing review engagements and the firm’s quality control procedures as well as minimize liability. The

extent of documentation will vary depending on the circumstances of the engagement, the methodology,

and tools used, as well as on the accountant’s professional judgment.

The following discussion first identifies the minimum documentation requirements included in SSARSs.

Accountants, however, are not precluded from exceeding these minimum requirements and including, for

example, written documentation contained in other engagement files or quality control files and, in

limited situations oral explanations. Accordingly, we have provided a discussion of the additional

documentation items we believe are necessary to support compliance with standards for review.

Required documentation

Documentation may be recorded on paper or on electronic or other media and does not need to include

replaced drafts of working papers, preliminary notes, or corrected or duplicate documents.

Documentation for review engagements should include the following:

The engagement letter, or written contract, documenting the understanding with the client. The engagement letter or contract should be signed by both the accountant and management or those charged with governance.

Communications, whether oral or written, to the appropriate level of management regarding fraud or illegal acts that come to the accountant’s attention.

Communications with management regarding the accountant’s expectation to include an emphasis-of-matter or other-matter paragraph in the accountant’s review report.

Communications with other accountants who have audited or reviewed the financial statements of significant components.

A copy of the reviewed financial statements and the accountant’s review report. The analytical procedures performed, including the following:

— The expectations, when the expectations are not otherwise readily determinable from the documentation of the work performed, and the factors considered in the development of the expectations.

— Results of the comparison of the expectations to the recorded amounts or ratios developed from recorded amounts.

— Management’s responses to the accountant’s inquiries regarding fluctuations or relationships that are inconsistent with other relevant information or that differ from expected values by a significant amount.

Any additional review procedures performed in response to significant unexpected differences arising from analytical procedures and the results of such additional procedures.

The significant matters covered in the accountant’s inquiry procedures and management’s responses to those inquiries.

Page 63: Performing Preparation, Compilation, and Review Engagements

© 2021 Association of International Certified Professional Accountants. All rights reserved. 55

Any findings or issues that, in the accountant’s judgment are significant (for example, the results of review procedures that indicate the financial statements could be materially misstated, including actions taken to address such findings, and the basis for the final conclusions reached).

Significant unusual matters that the accountant considered during the performance of the review procedures, including their disposition.

The management representation letter.

Documentation of knowledge of industry and business

Typically, documentation of the accountant’s knowledge of the industry and business is captured in a

client acceptance form that can be filed in the permanent file or the latest working paper file. The form,

which should be updated at least annually, notes the relevant industry and associated industry guides for

the client, and answers the basic questions about the client business.

The client acceptance form highlights information similar to that found on an audit planning form. The

use of a “risk approach” for a review engagement helps to identify financial statement captions that will

require special attention or adjustments and to focus the documentation on critical considerations. (It is

important to note, however, that risk assessment procedures like those required in auditing are not

required for review engagements. Instead, we recommend the risk approach as a tool practitioners can

use to maximize both efficiency and effectiveness.) Information concerning unique accounting principles

and practices, related parties, and so on, should be cross-referenced to the disclosure checklist

(discussed in a subsequent section) and requisite report modifications.

Review engagement program

A review engagement program outlines the engagement approach and identifies other forms and

checklists to be used on the engagement. Different firms will tailor the nature and extent of

documentation required by the program according to the uniqueness of their operations, their size, and

the complexity of their engagements. Although a program is not required by the standards governing

review engagements, we believe that a program should be completed for all review engagements.

Each of the major steps in a review engagement should be included in the program. Some of these steps

are completed in the planning phase by managers or partners. Others relate directly to the staff’s

obligations during the engagement. Note that the level of staffing may differ by firm and engagement.

The major steps include the following:

Make an acceptance or continuance decision. This step is usually completed by the partner or manager and documented on a Client Acceptance Form.

Consider whether the firm is independent. If the firm is not independent, the accountant will not be able to issue a review report.

Document any step-down considerations. This step is usually completed by the partner or manager. If the engagement had initially been an audit, the partner or manager must consider whether it is appropriate to step down to a lower level of service and document his or her conclusion.

Page 64: Performing Preparation, Compilation, and Review Engagements

© 2021 Association of International Certified Professional Accountants. All rights reserved. 56

Establish an understanding with the client. Engagement letters, or other suitable form of written contract, are required for review engagements and should be signed by the accountant and the client.

Determine scheduling and staff requirements. This step is usually completed by the partner or manager. Formal budgets are not required.

Communicate with other accountants. If significant components of the entity are reviewed or audited by others, the accountant should obtain reports from those other accountants as a basis for his or her review report. The accountant may also want to obtain a written confirmation of the independence of the other accountants, although it is not required that he or she do so.

Obtain knowledge of the industry. This step is typically documented on the client acceptance form, as discussed previously.

Obtain knowledge of the client. This step is typically documented on the client acceptance form, as discussed previously.

Consider the necessity of performing any other accounting services. Note that if the accountant has also performed a preparation or compilation engagement of the financial statements being reviewed, those preparation or compilation working papers should be referenced in the review working papers.

Determine materiality. Perform inquiry and analytical procedures. Obtain revised or additional information, if necessary. Draft the accountant’s report. Plan on using a disclosure checklist as discussed in a subsequent section. Obtain a representation letter. The management representation letter should include all periods

covered in the review report and should be signed by appropriate members of management. Read the financial statements. Ensure that the financial statements are free from obvious error. Utilize

the disclosure checklist discussed in a subsequent section. Document any communication of suspected fraud or illegal acts. Communicate any suspected fraud

or illegal acts, unless they are clearly inconsequential. Document consultation, resolve professional disputes, clear open items, and review notes.

Review working papers. The in-charge should document his or her review of the forms, checklists, and working papers.

Consider passed adjustments. These items should be considered after the normal closing entries have been booked. The items should be evaluated individually and in the aggregate in relation to account balances, subtotals, or totals in the financial statements. Roll forward passed adjustments from prior years’ engagements and document consideration of the effect of analytical procedures performed during the course of the review engagement.

Engagement partner review and approval. This approval indicates that the financial statements are appropriate in form and free from obvious material misstatement. The approval also indicates that the report form is appropriate, and that the financial statements have been prepared, reviewed, and processed in accordance with firm policies.

Disclosure checklist

All disclosures required by the applicable financial reporting framework should be included with the

reviewed financial statements, or their omission, if material, should be disclosed in the review report.

Because the list of possible disclosures is typically very long, and the risk of inadvertently omitting a

required disclosure can be high, we recommend using a disclosure checklist developed for non-issuers

(for example, those that do not have listed securities, filings with a regulatory agency in preparation for a

public sale of securities or a subsidiary, or are joint ventures or controlled enterprises of a public entity).

Page 65: Performing Preparation, Compilation, and Review Engagements

© 2021 Association of International Certified Professional Accountants. All rights reserved. 57

Support for financial statement notes

In addition to ensuring that all necessary notes are included with the financial statements, working paper

documentation should include support for the notes to the financial statements.

Knowledge check

4. Which of the following is not required documentation for a review engagement?

a. Management’s responses to inquiries regarding the effectiveness of the client’s internal control system.

b. The signed engagement letter. c. Additional review procedures performed in response to significant unexpected differences

arising from analytical procedures. d. A management representation letter.

Page 66: Performing Preparation, Compilation, and Review Engagements

© 2021 Association of International Certified Professional Accountants. All rights reserved. 58

Change in level of service On occasion, an accountant engaged to audit the financial statements of a non-issuer in accordance with

GAAP may receive a request to change the engagement to a review of financial statements. This request

to “step-down” in service should not be treated lightly or automatically granted by the accountant. A

change in circumstances that affects the entity’s requirement for an audit or a misunderstanding

concerning the nature of an audit or review would ordinarily be considered a reasonable basis for

requesting a change in the engagement.

If a change in circumstances or misunderstanding is not involved, the accountant should consider

several issues. First, the accountant should consider the reason given for the client’s request, particularly

the implication of a restriction on the scope of the audit, whether imposed by the client or circumstances.

In considering the implications of a restriction on the scope of the audit, the accountant should evaluate

the possibility that information affected by the scope restriction may be incorrect, incomplete, or

otherwise unsatisfactory. That said, however, when the accountant has been prohibited by the client

from corresponding with the entity’s legal counsel, the accountant would normally be precluded from

issuing a review report and accordingly, could not step-down the engagement.

Second, the accountant should consider both the additional audit effort required to complete the audit

and the estimated additional cost to complete the audit. If the audit procedures are substantially

complete or the cost to complete the procedures is relatively insignificant, the accountant should

consider the propriety of accepting a change in engagement.

If the accountant concludes that there is a reasonable justification to change the engagement, and the

accountant complies with standards applicable to a review engagement, the appropriate review report

may be issued. The report should not, however, refer to the original engagement, any audit procedures

performed, or scope limitations that resulted in the changed engagement.

Knowledge check

5. What would an accountant who is asked to step down the level of service from an audit to a review engagement do?

a. Consider the additional effort needed to complete the audit. b. Not consider the reason for the client’s request. c. Not consider the estimated cost to complete the audit. d. Consider the effect on the auditor’s opinion.

Page 67: Performing Preparation, Compilation, and Review Engagements

© 2021 Association of International Certified Professional Accountants. All rights reserved. 59

Summary We have just discussed the following points:

What constitutes a review engagement The performance requirements for a review engagement The documentation requirements of a review engagement The forms and checklists used in a review engagement Changes in level of service

Page 68: Performing Preparation, Compilation, and Review Engagements

© 2021 Association of International Certified Professional Accountants. All rights reserved. 60

Practice questions and case studies 1. Fill in the following chart that compares review and audit engagements.

Review engagement Audit engagement

1. Level of assurance

2. Entities covered

3. Knowledge of the client and their

industry

4. Inquiry and analytical procedures

required

5. Disclosures required by the applicable

financial reporting framework are

omitted

6. Known departures from applicable

financial reporting framework

7. Accountant’s independence

8. Obtain an understanding of internal

control

9. Engagement letter

10. Management representation letter

2. Identify the performance requirements for a review engagement.

3. Are analytical procedures performed in a review different from those in an audit?

4. What distinguishes working papers prepared for a review from those prepared in an audit?

Page 69: Performing Preparation, Compilation, and Review Engagements

© 2021 Association of International Certified Professional Accountants. All rights reserved. 61

5. What distinguishes the accountant’s responsibility for accounting estimates in a review from that of an audit?

Case: Client acceptance

Martha Fraser of Fraser, Giddens, and Howard LLP (FGH) is approached by Charles Bonner about providing a review engagement for the financial statements of Bonner Pharmaceuticals. Bonner is seeking the review engagement to include it in a proposal seeking funding from prospective investors and creditors.

Consider each of the following (independent) pieces of information that may become available while FGH is considering whether to accept the review engagement. How should the information affect FGH’s decision? What additional information might FGH need to determine if it will accept Bonner as a client?

1. Another client informs Martha that Charles told prospective investors and creditors that reviewed financial statements are the same as an audited set of financial statements.

2. Martha and FGH currently have no clients in the pharmaceutical industry. 3. Bonner Pharmaceuticals plans to use the tax basis of accounting to prepare its financial

statements.

Case: Analytical procedures

Your firm, Staid & Company, has recently been engaged by New Idea Co., a management consulting firm, to review their year-end financial statements. The staff accountant who performed the engagement, Christine Powerthru, has recently joined your staff after working for three years for another area CPA firm, Focused & Associates. Focused & Associates limits their practice to tax and compilations, and is noted for its extensive use of technology, in particular, cloud computing. As part of engagement partner review of this engagement, the following facts come to your attention.

New Idea Co. has been in existence for 24 months and the financial statements are for the most recent 12-month period. New Idea Co. has grown from two staff members to over 100 employees primarily through mergers and acquisitions financed through bank borrowings. Formal financial statements were not prepared at the end of the first year of operations. One of New Idea Co.’s shareholders, Lee B. Fast, kept the books using a well-known accounting software package and prepared the tax return. Lee’s accounting training consists of one semester of high school bookkeeping and the instructions that came with the software package.

Page 70: Performing Preparation, Compilation, and Review Engagements

© 2021 Association of International Certified Professional Accountants. All rights reserved. 62

Case: Analytical procedures (continued)

Staid & Company was engaged two months prior to the current year end to provide monthly compilation services and the year-end review and tax work. The monthly financial statements had not been compiled for any month in the current year. However, a trial balance prepared by Lee was available for the first 10 months of the year. Christine supervised the paraprofessional staff during the input into Staid’s software of the 10-month balances and the two remaining months, as well as preparation of the year-end trial balance. Christine went over the recommended year-end adjustments with the client who approved each adjustment before the year-end financial statements were printed. Interim financial statements were not prepared for any period.

During your review of the analytical procedures worksheet you notice that Christine diligently computed each ratio available with the review software. The ratios were computed from the current year financial statements and from last year's tax return. Christine is the first accountant at Staid & Company to download the information directly from the cloud into review software. Christine also downloaded the prior year balances directly from Lee's program to the review software. In addition, Christine searched for industry ratios and located average data for the top 10 management consulting firms in the United States in a recent issue of Business Week. Christine wrote a comment for every ratio calculated.

Required

Comment on the analytical procedures Christine performed and, if you deem it necessary, make recommendations about the course of action to complete the engagement.

Case: Review evidence

Kendra Wagner is working on a review engagement for Durham Construction Company. In the past couple of years, Durham Construction has felt the impact of an economic downturn, and as a result the owners hired a new president last year. Despite changes in other areas of the company related to the economic downturn, the accounting department has experienced very little turnover. Kendra’s experience with the management of Durham has indicated that they are honest with a great understanding of their business and clients. She is not convinced, however, that they have an equally great understanding of some of their accounting issues. Assuming that each of the following situations is material to the financial statements, what, if anything, should Kendra do to wrap up this engagement?

Page 71: Performing Preparation, Compilation, and Review Engagements

© 2021 Association of International Certified Professional Accountants. All rights reserved. 63

Case: Review evidence (continued)

1. Accounts receivable has increased by 28% compared to the prior year, without any increase in the allowance for doubtful accounts. When asked, the new president stated he had studied the allowance extensively and Durham has been more restrictive in extending credit, so he still expects uncollectible accounts to be less than the company has reserved.

2. Property tax expense is slightly lower than in the prior year, even though Durham bought new land and a new building and had no disposals. The president told Kendra that there are no real estate taxes on the newly acquired land and building until next year.

3. The president tells Kendra that a material uninsured lawsuit has been filed against Durham. The president believes that it is frivolous, and accordingly no provision has been made in the financial statements for it.

Page 72: Performing Preparation, Compilation, and Review Engagements

© 2021 Association of International Certified Professional Accountants. All rights reserved. 64

Page 73: Performing Preparation, Compilation, and Review Engagements

© 2021 Association of International Certified Professional Accountants. All rights reserved. 65

Solutions

Performing Review Engagements

Knowledge check solutions

1.

a. Incorrect. The purpose of an audit engagement is to provide reasonable assurance on the financial statements.

b. Correct. The purpose of a review engagement is to provide limited assurance on the financial statements.

c. Incorrect. A review engagement does provide some assurance on the financial statements.

d. Incorrect. No engagement provides absolute assurance on financial statements.

2.

a. Correct. A review engagement does not contemplate assessing fraud risk.

b. Incorrect. Accountants performing a review engagement are not required to search for fraud or illegal acts.

c. Incorrect. The accountant is required to have enough knowledge of the industry to complete the engagement. Expert knowledge is not required.

d. Incorrect. The accountant should communicate evidence of fraud or illegal acts, unless they are clearly inconsequential, to management or those charged with governance.

3.

a. Incorrect. If information is inaccurate, incomplete, or otherwise unsatisfactory, the accountant will need to follow up, and that may require additional procedures.

b. Correct. The work performed to obtain enough review evidence may be tailored to the client, taking into consideration the industry characteristics (for example, such as degree of regulation) and business characteristics (for example, size of organization).

c. Incorrect. Both an engagement and a management representation letter are required in review engagements.

d. Incorrect. An accountant is not required to understand or test internal controls in order to obtain the limited assurance for a review engagement.

Page 74: Performing Preparation, Compilation, and Review Engagements

© 2021 Association of International Certified Professional Accountants. All rights reserved. 66

4.

a. Correct. Understanding and testing the client’s internal control system are not requirements for a review engagement. Accordingly, accountants are not required to include the client’s internal control system in their inquiries of management, and such inquiries are not part of the required documentation of a review engagement.

b. Incorrect. Accountants must include an engagement letter signed by both parties in documentation of a review engagement.

c. Incorrect. Accountants may be required to perform additional review procedure in response to significant unexpected results of analytical procedures. If the accountant performs those procedures, he or she must include them in the documentation of the review engagement.

d. Incorrect. A signed management representation letter is required documentation for a review engagement.

5.

a. Correct. The accountant should consider the additional effort needed to complete an audit.

b. Incorrect. The accountant should consider the legitimacy of the client’s request to step down from an audit to a review engagement.

c. Incorrect. The accountant should consider the estimated costs to complete the audit.

d. Incorrect. If the accountant steps the engagement down to a review, there will be no auditor’s opinion. If the accountant provides an audit, there will be no mention of the client’s request in the auditor’s opinion.

Practice question solutions

1.

Review engagement Audit engagement

1. Level of assurance Limited as to U.S. generally accepted accounting principles (GAAP) or special purpose framework (SPF)

Reasonable Assurance - Statements are fairly presented in accordance with GAAP or SPF

2. Entities covered Non-issuers Issuers or non-issuers

3. Knowledge of the client and their industry

Same as compilation plus an increased knowledge of the client’s business

Extensive knowledge of the economy, the relevant industry, and the client’s business

Page 75: Performing Preparation, Compilation, and Review Engagements

© 2021 Association of International Certified Professional Accountants. All rights reserved. 67

Review engagement Audit engagement

4. Inquiry and analytical procedures required

Inquiry and analytical procedures required plus additional procedures if the information appears questionable

Inquiry, analytical procedures, and other audit procedures required

5. Disclosures required by the applicable financial reporting framework are omitted

All disclosures required by the applicable financial reporting framework are required to be included

Inadequate disclosure requires qualified or adverse opinion

6. Known departures from the applicable financial reporting framework

Disclosure is required in modified report

Requires qualified or adverse opinion

7. Accountant’s independence

Lack of independence precludes issuing report

Lack of independence precludes issuing report – must issue a disclaimer

8. Obtain an understanding of internal control

Not required

Required

9. Engagement letter Required

Required

10. Management representation letter

Required Required

2. The performance requirements for review engagements are as follows:

Establish an understanding with the client Communicate with management or those charged with governance

Have (or obtain) knowledge of the industry that is sufficient to enable the practitioner to construct inquiries and analytical procedures adequate to provide a reasonable basis for expressing limited assurance

Have (or obtain) knowledge of the client’s business transactions, accounting records, personnel, accounting basis, and form and content of the financial statements that is sufficient to enable the practitioner to construct inquiries and analytical procedures adequate to provide a reasonable basis for expressing limited assurance

Obtain sufficient appropriate review evidence

Reconcile financial statements to underlying records Perform inquiry and analytical procedures

Obtain additional information if the practitioner becomes aware that the information supplied is incorrect, incomplete, or otherwise unsatisfactory

Obtain a management representation letter

Read the financial statements to ensure that they are free from obvious errors

Page 76: Performing Preparation, Compilation, and Review Engagements

© 2021 Association of International Certified Professional Accountants. All rights reserved. 68

3. Although the nature of the analytical procedures may be the same (for example, comparison of current and prior year’s amounts, ratio analysis, and so on) the nature and extent of follow-up work on the analytical procedures will differ. Because a review is designed to provide limited assurance, accountants may not obtain the same quality or quantity of corroborating evidence about an account as they would in an audit.

4. Working papers prepared in a review engagement document the accountant’s knowledge of the client’s business and industry. The analyses support the analytical procedures and the inquiries performed, including an identification of the individuals responding to the inquiries. In addition, the working papers document unusual matters that the accountant considered during the performance of the review, including disposition.

Audit working papers document the audit procedures applied, evidence obtained, and conclusions reached. The working papers provide corroborating evidence that (1) the audit has been properly planned and supervised; (2) the nature, timing, and extent of audit procedures have been determined after a consideration of engagement risk, inherent risk, and control risk; and (3) there is an appropriate level of risk of material undetected misstatements in the financial statement assertions.

Because the purpose of working papers in a review differs from that of an audit, the form and content of the documentation should as well.

5. The auditor is required to obtain sufficient competent evidence to provide reasonable assurance that all accounting estimates material to the financial statements have been developed by

management. — Identify circumstances requiring accounting estimates.

those estimates are reasonable. — Review and test the process used by management to develop the estimates,

including the control structure, sources, and reliability of data and assumptions. — Develop an independent expectation to corroborate management’s estimate. — Review subsequent events or transactions.

Due professional care requires the investigation of differences between an accounting estimate best supported by audit evidence and the estimated amounts included in the financial statements.

AR-C section 90 does not require the accountant to

obtain corroborating evidential matter by testing accounting records or responses to inquiries.

obtain an understanding of the internal control structure.

Page 77: Performing Preparation, Compilation, and Review Engagements

© 2021 Association of International Certified Professional Accountants. All rights reserved. 69

Case: Client acceptance solutions

1. According to Statements on Standards for Accounting and Review Services (SSARSs), accountants should typically not accept or continue client relationships in which the accountant has cause to doubt management’s integrity such that it is likely to affect the performance of the engagement.

2. The AICPA Code of Professional Conduct requires that an accountant should have sufficient professional competence to undertake an engagement, so it is important for FGH to consider its lack of expertise. SSARSs notes, however, that accountants are not required to have appropriate knowledge of the industry at the time a review engagement is accepted. Instead, the engagement can be accepted with the understanding that an appropriate understanding of the industry will be obtained. The issue, then, is the degree to which FGH can obtain appropriate expertise and training. FGH must consider whether it is willing to spend the time and effort to train its staff, as well as whether it has sufficient resources (both time and funds) to train the staff.

3. The tax basis of accounting is a special purpose framework (SPF) appropriate for use by Bonner Pharmaceuticals. Prior to accepting the engagement, FGH will need to get management to agree that it acknowledges its responsibility for preparing and presenting the financial statements in accordance with the tax basis of accounting and including all informative disclosures necessary, including

a. a description of the tax basis of accounting, including a summary of significant accounting policies and how the tax basis differs from U.S. generally accepted accounting principles (GAAP), as well as disclosures similar to those required by GAAP.

b. any additional disclosures beyond that may be necessary for the tax basis of accounting to achieve fair presentation.

Case: Analytical procedures solution

The purpose of the review software is to provide computational assistance for the typical liquidity, profitability, activity, and leverage ratios. Not all ratios are applicable for each review engagement, and only those that are applicable should be computed. The choice of ratios should be based on the accountant’s knowledge of the client’s business and industry. Judgment should be exercised in determining which procedures to perform. In some instances, inquiries may be more efficient than analytical procedures. Unnecessary analytical procedures, or any other unnecessary procedure, should be avoided because they add cost to the engagement. Further, if the procedure indicates a deviation, the deviation should be explained. In addition, the unnecessary procedure might lead to increased risk to the CPA firm if the firm is the defendant in a suit related to this client.

Christine was correct to try and compare the results of the analytical procedures with the entity’s past performance and with industry averages. However, the comparisons are concerning. The industry averages used for comparative purposes were for the leading 10 firms

Page 78: Performing Preparation, Compilation, and Review Engagements

© 2021 Association of International Certified Professional Accountants. All rights reserved. 70

in the industry. It is doubtful that the 10 leading firms are valid in terms of size and other characteristics. Another source with relevant data should be utilized.

Christine’s comparison to prior year client data is also flawed. The prior year is the initial year of operations for the client. It is doubtful that comparisons, particularly for the income statement, are appropriate. In addition, there is the underlying question of data integrity. The prior year numbers and 5/6 of the current year work were prepared by someone whose accounting knowledge is limited. Review of the prior year ending balances and the January–October work for this year should be accomplished. Any indicated adjustments should be made prior to analytical procedures.

Professional judgment should be used to determine the definition of significant fluctuations from expected amounts. The determination should be based on the expected size of the fluctuation, materiality, percentage change, and the precision of the particular analytical procedure. When the results of the analytical procedures indicate significant fluctuations, the accountant is required to perform additional procedures. These additional procedures may include additional analytical procedures, inquiries, and detailed account analysis. Although Christine wrote a comment on each ratio, there is no indication that those that require further investigation were indicated. Once these data are deemed valid and the ratios computed again, Christine should communicate those ratios that require further investigation.

Case: Review evidence solutions

1. The significant increase in receivables without a corresponding increase in collectibility raises issues related to both existence and valuation. A good starting place would be to review and verify the new credit procedures described by the president as well as his extensive analysis of the allowance account. If Kendra’s manager is not satisfied with the results of these procedures, Kendra could also consider verifying the existence of significant or unusual sales in the sales ledger.

2. There is little about the new president’s explanation that sounds accurate. Kendra should consider reviewing the closing documents on the new property to determine whether there are some unusual terms related to property taxes. She may also consider reviewing tax bills on existing property to ensure that all property taxes have been appropriately included.

3. This is a question for an attorney. At a minimum, Kendra should contact Durham’s counsel and inquire about the lawsuit. Unlike an audit, there is no requirement that this communication be in writing. However, the authors suggest written communication.

Page 79: Performing Preparation, Compilation, and Review Engagements

© 2021 Association of International Certified Professional Accountants. All rights reserved. 71

AR-C Glossary analytical procedures. Evaluations of financial information through analysis of plausible

relationships among both financial and nonfinancial data. Analytical procedures also encompass such investigation, as is necessary, of identified fluctuations or relationships that are inconsistent with other relevant information or that differ from expected values by a significant amount.

applicable financial reporting framework. The financial reporting framework adopted by management and, when appropriate, those charged with governance, in the preparation and fair presentation of the financial statements that is acceptable in view of the nature of the entity and the objective of the financial statements or that is required by law or regulation.

basic financial statements. Financial statements excluding supplementary information and required supplementary information.

comparative financial statements. A complete set of financial statements for one or more prior periods included for comparison with the financial statements of the current period.

designated accounting standard-setter. A body designated by the Council of the AICPA to promulgate accounting principles generally accepted in the United States of America pursuant to the “Compliance With Standards Rule” (ET sec. 1.310.001) and the “Accounting Principles Rule” (ET sec. 1.320.001) of the AICPA Code of Professional Conduct.

emphasis-of-matter paragraph. A paragraph included in the accountant’s review report that is required by Statements on Standards for Accounting and Review Services (SSARSs), or is included at the accountant’s discretion, and that refers to a matter appropriately presented or disclosed in the financial statements that, in the accountant’s professional judgment, is of such importance that it is fundamental to the users’ understanding of the financial statements.

engagement partner.1 The partner or other person in the firm who is responsible for the engagement and its performance and for the report that is issued on behalf of the firm and who, when required, has the appropriate authority from a professional, legal, or regulatory body.

engagement team. All accountants and staff performing the engagement and any individuals engaged by the firm who perform procedures on the engagement.

error. Mistakes in the financial statements, including arithmetical or clerical mistakes, and mistakes in the application of accounting principles, including inadequate disclosures.

1 This term is also defined in paragraph .13 of QC section 10, A Firm’s System of Quality Control, for purposes of the

Statements on Quality Control Standards. Refer to QC section 10 for specific language.

Page 80: Performing Preparation, Compilation, and Review Engagements

© 2021 Association of International Certified Professional Accountants. All rights reserved. 72

experienced accountant. An individual (whether internal or external to the firm) who has practical review experience and a reasonable understanding of

a. review processes; b. SSARSs and applicable legal and regulatory requirements; c. the business environment in which the entity operates; and d. review and financial reporting issues relevant to the entity’s industry.

financial reporting framework. A set of criteria used to determine measurement, recognition, presentation, and disclosure of all material items appearing in the financial statements (for example, accounting principles generally accepted in the United States of America [U.S. GAAP], International Financial Reporting Standards promulgated by the International Accounting Standards Board, or a special purpose framework).

financial statements. A structured representation of historical financial information, including related notes, intended to communicate an entity’s economic resources and obligations at a point in time or the changes therein for a period of time in accordance with a financial reporting framework. The related notes ordinarily comprise a summary of significant accounting policies and other explanatory information. The term financial statements ordinarily refers to a complete set of financial statements as determined by the requirements of the applicable financial reporting framework but can also refer to a single financial statement.

firm. A form of organization permitted by law or regulation whose characteristics conform to resolutions of the Council of the AICPA and that is engaged in the practice of public accounting.

fraud. An intentional act that results in a misstatement in financial statements.

generally accepted accounting principles (GAAP). References to generally accepted accounting principles in SSARSs means generally accepted accounting principles promulgated by bodies designated by the Council of the AICPA pursuant to the “Compliance With Standards Rule” (ET sec. 1.310.001) and the “Accounting Principles Rule” (ET sec. 1.320.001) of the AICPA Code of Professional Conduct.

historical financial information. Information expressed in financial terms regarding a particular entity, derived primarily from that entity’s accounting system, about economic events occurring in past time periods or about economic conditions or circumstances at points in time in the past.

interpretive publications. Interpretations of SSARSs; exhibits to SSARSs; the AICPA Audit & Assurance Guide Preparation, Compilation, and Review Engagements, guidance on reviews, compilations, and engagements to prepare financial statements included in AICPA Audit and Accounting Guides; and Statements of Position, to the extent that those statements are applicable to such engagements.

management. The person(s) with executive responsibility for the conduct of the entity’s operations. For some entities, management includes some or all of those charged with governance (for example, executive members of a governance board or an owner-manager).

Page 81: Performing Preparation, Compilation, and Review Engagements

© 2021 Association of International Certified Professional Accountants. All rights reserved. 73

misstatement. A difference between the amount, classification, presentation, or disclosure of a reported financial statement item and the amount, classification, presentation, or disclosure that is required for the item to be presented fairly in accordance with the applicable financial reporting framework. Misstatements can arise from fraud or error. Misstatements also include those adjustments of amounts, classifications, presentations, or disclosures that, in the accountant’s professional judgment, are necessary for the financial statements to be presented fairly, in all material respects.

noncompliance. Acts of omission or commission by the entity, either intentional or unintentional, which are contrary to the prevailing laws or regulations. Such acts include transactions entered into, by, or in the name of, the entity or on its behalf by those charged with governance, management, or employees. Noncompliance does not include personal misconduct (unrelated to the business activities of the entity) by those charged with governance, management, or employees of the entity.

other-matter paragraph. A paragraph included in the accountant’s review report that is required by SSARSs, or is included at the accountant’s discretion, and that refers to a matter other than those presented or disclosed in the financial statements that, in the accountant’s professional judgment, is relevant to users’ understanding of the review, the accountant’s responsibilities, or the accountant’s review report.

other preparation, compilation and review publications. Publications other than interpretive publications. These include AICPA accounting and review publications not defined as interpretive publications; the annual Developments in Review, Compilation, and Financial Statement Preparation Engagements Alert; articles addressing reviews, compilations, and engagements to prepare financial statements in the Journal of Accountancy and other professional journals; continuing professional education programs and other instruction materials, textbooks, guide books, programs for reviews, compilations, and engagements to prepare financial statements, and checklists; and other publications addressing reviews, compilations, and engagements to prepare financial statements from state CPA societies, other organizations, and individuals.

professional judgment. The application of relevant training, knowledge, and experience, within the context provided by SSARSs and ethical standards, in making informed decisions about the courses of action that are appropriate in the circumstances of the review, compilation, or engagement to prepare financial statements.

report release date. The date the accountant grants the entity permission to use the accountant’s review report in connection with the financial statements.

required supplementary information. Information that a designated accounting standards-setter requires to accompany an entity’s basic financial statements. Required supplementary information is not part of the basic financial statements; however, a designated accounting standards-setter considers the information to be an essential part of financial reporting for placing the basic financial statements in an appropriate operational, economic, or historical context. In addition, authoritative guidelines for the methods of measurement and presentation of the information have been established.

Page 82: Performing Preparation, Compilation, and Review Engagements

© 2021 Association of International Certified Professional Accountants. All rights reserved. 74

review documentation. The record of review procedures performed, relevant review evidence obtained, and conclusions the accountant reached (terms such as working papers or workpapers are also sometimes used).

review evidence. Information used by the accountant to provide a reasonable basis for obtaining limited assurance.

special purpose framework. A financial reporting framework other than GAAP that is one of the following bases of accounting:

a. Cash basis. A basis of accounting that the entity uses to record cash receipts and disbursements and modifications of the cash basis having substantial support (for example, recording depreciation on fixed assets).

b. Tax basis. A basis of accounting that the entity uses to file its tax return for the period covered by the financial statements.

c. Regulatory basis. A basis of accounting that the entity uses to comply with the requirements or financial reporting provisions of a regulatory agency to whose jurisdiction the entity is subject (for example, a basis of accounting that insurance companies use pursuant to the accounting practices prescribed or permitted by a state insurance commission).

d. Contractual basis. A basis of accounting that the entity uses to comply with an agreement between the entity and one or more third parties other than the accountant.

e. Other basis. A basis of accounting that uses a definite set of logical, reasonable criteria that is applied to all material items appearing in financial statements.

The cash-basis, tax-basis, regulatory-basis, and other-basis of accounting are commonly referred to as other comprehensive bases of accounting (OCBOA).

specified parties. The intended users of the accountant’s review report.

subsequent events. Events occurring between the date of the financial statements and the date of the accountant’s review report.

subsequently discovered facts. Facts that become known to the accountant after the date of the accountant’s review report that, had they been known to the accountant at that date, may have caused the accountant to revise the accountant’s review report.

supplementary information. Information presented outside the basic financial statements, excluding required supplementary information, that is not considered necessary for the financial statements to be fairly presented in accordance with the applicable financial reporting framework.

those charged with governance. The person(s) or organization(s) (for example, a corporate trustee) with responsibility for overseeing the strategic direction of an entity and the obligations related to the accountability of the entity. This includes overseeing the financial reporting process. Those charged with governance may include management personnel (for example, executive members of a governance board or an owner-manager).

Page 83: Performing Preparation, Compilation, and Review Engagements

© 2021 Association of International Certified Professional Accountants. All rights reserved. 75

updated report. A report issued by a continuing accountant that takes into consideration information that the accountant becomes aware of during the accountant’s current engagement and that re-expresses the accountant’s previous conclusions or, depending on the circumstances, expresses different conclusions on the financial statements of a prior period reviewed by the accountant as of the date of the accountant’s current report.

written representation. A written statement by management provided to the accountant to confirm certain matters or to support other review evidence. Written representations in this context do not include financial statements, the assertions therein, or supporting books and records.

Page 84: Performing Preparation, Compilation, and Review Engagements

© 2021 Association of International Certified Professional Accountants. All rights reserved. 76

Page 85: Performing Preparation, Compilation, and Review Engagements

© 2021 Association of International Certified Professional Accountants. All rights reserved. 77

Accounting and Auditing Glossary account. Formal record that represents, in words, money or other unit of measurement, certain

resources, claims to such resources, transactions or other events that result in changes to those resources and claims.

account payable. Amount owed to a creditor for delivered goods or completed services.

account receivable. Claim against a debtor for an uncollected amount, generally from a completed transaction of sales or services rendered.

accountants’ report. Formal document that communicates an independent accountant’s (1) expression of limited assurance on financial statements as a result of performing inquiry and analytic procedures (Review Report); (2) results of procedures performed (type of Attestation Report); (3) non-expression of opinion or any form of assurance on a presentation in the form of financial statements information that is the representation of management (Compilation Report); or (4) an opinion on an assertion made by management in accordance with the Statements on Standards for Attestation Engagements (Attestation Report). An accountant’s report does not result from the performance of an audit.

accounting. Recording and reporting of financial transactions, including the origination of the transaction, its recognition, processing, and summarization in the financial statements.

accounting change. Change in (1) an accounting principle; (2) an accounting estimate; or (3) the reporting entity. The correction of an error in previously issued financial statements is not an accounting change.

accrual basis. Method of accounting that recognizes revenue when earned, rather than when collected. Expenses are recognized when incurred rather than when paid.

accrued expense. An expense incurred during an accounting period for which payment is not due until a later accounting period. This results from the purchase of services which at the time of accounting have only been partly performed, are not yet billable, or have not been paid for.

accumulated depreciation. Total depreciation pertaining to an asset or group of assets from the time the assets were placed in service until the date of the financial statement or tax return. This total is the contra account to the related asset account.

additional paid in capital. Amounts paid for stock in excess of its par value or stated value. Also, other amounts paid by stockholders and charged to equity accounts other than capital stock.

adjusting entries. Accounting entries made at the end of an accounting period to allocate items between accounting periods.

amortization. The process of reducing a recognized liability systematically by recognizing revenues or by reducing a recognized asset systematically by recognizing expenses or costs.

Page 86: Performing Preparation, Compilation, and Review Engagements

© 2021 Association of International Certified Professional Accountants. All rights reserved. 78

In accounting for postretirement benefits, amortization also means the systematic recognition in net periodic postretirement benefit cost over several periods of amounts previously recognized in other comprehensive income, that is, gains or losses, prior service cost or credits, and any transition obligation or asset.

analytical procedures. Substantive tests of financial information which examine relationships among data as a means of obtaining evidence. Such procedures include (1) comparison of financial information with information of comparable prior periods; (2) comparison of financial information with anticipated results (e.g., forecasts); (3) study of relationships between elements of financial information that should conform to predictable patterns based on the entity’s experience; and (4) comparison of financial information with industry norms.

annual report. The annual report to shareholders is the principal document used by most public companies to disclose corporate information to their shareholders. It is usually a state-of-the-company report, including an opening letter from the Chief Executive Officer, financial data, results of continuing operations, market segment information, new product plans, subsidiary activities, and research and development activities on future programs. The Form 10-K, which must be filed with the SEC, typically contains more detailed information about the company’s financial condition than the annual report.

assertion. Explicit or implicit representations by an entity’s management that are embodied in financial statement components and for which the auditor obtains and evaluates evidential matter when forming his/her opinion on the entity’s financial statements.

audit risk. The risk that the auditor may unknowingly fail to modify appropriately his/her opinion on financial statements that are materially misstated.

audit sampling. Application of an audit procedure to less than 100% of the items within an account balance or class of transactions for the purpose of evaluating some characteristic of the balance or class.

auditors’ report. Written communication issued by an independent certified public accountant (CPA) describing the character of his/her work and the degree of responsibility taken. An auditor’s report includes a statement that the audit was conducted in accordance with generally accepted auditing standards (GAAS), which require that the auditor plan and perform the audit to obtain reasonable assurance about whether the financial statements are free of material misstatement, as well as a statement that the auditor believes the audit provides a reasonable basis for his/her opinion.

bad debt. All or portion of an account, loan, or note receivable considered to be uncollectible.

balance sheet. Basic financial statement, usually accompanied by appropriate disclosures that describe the basis of accounting used in its preparation and presentation of a specified date the entity’s assets, liabilities, and the equity of its owners. Also known as a statement of financial condition.

Page 87: Performing Preparation, Compilation, and Review Engagements

© 2021 Association of International Certified Professional Accountants. All rights reserved. 79

bond. One type of long-term promissory note, frequently issued to the public as a security regulated under federal securities laws or state blue sky laws. Bonds can either be registered in the owner’s name or are issued as bearer instruments.

book value. Amount, net or contra account balances, that an asset or liability shows on the balance sheet of a company. Also known as carrying value.

business combinations. Combining of two entities. Under the purchase method of accounting, one entity is deemed to acquire another and there is a new basis of accounting for the assets and liabilities of the acquired company.

business segment. Any division of an organization authorized to operate, within prescribed or otherwise established limitations, under substantial control by its own management.

capital stock. Ownership shares of a corporation authorized by its articles of incorporation. The money value assigned to a corporation’s issued shares. The balance sheet account with the aggregate amount of the par value or stated value of all stock issued by a corporation.

capitalized cost. Expenditure identified with goods or services acquired and measured by the amount of cash paid or the market value of other property, capital stock, or services surrendered. Expenditures that are written off during two or more accounting periods.

carrying value. Amount, net or contra account balances, that an asset or liability shows on the balance sheet of a company. Also known as book value.

cash basis. A special purpose framework in which revenues and expenditures are recorded when they are received and paid.

cash equivalents. Short-term (generally less than three months), highly liquid investments that are convertible to known amounts of cash.

cash flows. Net of cash receipts and cash disbursements relating to a particular activity during a specified accounting period.

casualty loss. Sudden property loss caused by theft, accident, or natural causes.

change in engagement. A request, before the completion of the audit (review), to change the engagement to a review or compilation (compilation) of financial statements.

class actions. A federal securities class action is a court action filed on behalf of a group of shareholders under Rule 23 of the Federal Rules of Civil Procedure. Instead of each shareholder bringing an individual lawsuit, one or more shareholders bring a class action for the entire class of shareholders.

common stock. Capital stock having no preferences generally in terms of dividends, voting rights, or distributions.

companies, going public. Companies become public entities for different reasons, but usually to raise additional capital. The SEC has prepared a guide for companies – Q&A: Small Business and the SEC – that provides a basic understanding about the various ways companies can become public and what securities laws apply. The SEC also has a list of some of the

Page 88: Performing Preparation, Compilation, and Review Engagements

© 2021 Association of International Certified Professional Accountants. All rights reserved. 80

registration and reporting forms and related regulations that pertain to small and large companies.

comparative financial statement. Financial statement presentation in which the current amounts and the corresponding amounts for previous periods or dates also are shown.

compilation. Presentation in the form of financial statements information that is the representation of management (owners) without the accountant’s assurance as to conformity with generally accepted accounting principles (GAAP).

comprehensive income. Change in equity of a business entity during a period from transactions and other events and circumstances from nonowner sources. The period includes all changes in equity except those resulting from investments by owners and distributions to owners.

confirmation. Auditor’s receipt of a written or oral response from an independent third party verifying the accuracy of information requested.

consolidated financial statements. Combined financial statements of a parent company and one or more of its subsidiaries as one economic unit.

consolidation. The presentation of a single set of amounts for an entire reporting entity. Consolidation requires elimination of intra-entity transactions and balances.

contingent liability. Potential liability arising from a past transaction or a subsequent event.

continuing accountant. An accountant who has been engaged to audit, review, or compile and report on the financial statements of the current period and one or more consecutive periods immediately prior to the current period.

control risk. Measure of risk that errors exceeding a tolerable amount will not be prevented or detected by an entity’s internal controls.

controls tests. Tests directed toward the design or operation of an internal control structure policy or procedure to assess its effectiveness in preventing or detecting material misstatements in a financial report.

current asset. Asset that one can reasonably expect to convert into cash, sell, or consume in operations within a single operating cycle, or within a year if more than one cycle is completed each year.

current liability. Obligation whose liquidation is expected to require the use of existing resources classified as current assets, or the creation of other current liabilities.

current value. (1) Value of an asset at the present time as compared with the asset’s historical cost. (2) In finance, the amount determined by discounting the future revenue stream of an asset using compound interest principles.

debt. General name for money, notes, bonds, goods, or services which represent amounts owed.

Page 89: Performing Preparation, Compilation, and Review Engagements

© 2021 Association of International Certified Professional Accountants. All rights reserved. 81

definite criteria. A special purpose framework using a definite set of criteria having substantial support that is applied to all material items appearing in financial statements, such as the price-level basis of accounting.

depreciation. Expense allowance made for wear and tear on an asset over its estimated useful life.

derivatives. Derivatives are financial instruments whose performance is derived, at least in part, from the performance of an underlying asset, security or index. For example, a stock option is a derivative because its value changes in relation to the price movement of the underlying stock.

detection risk. Risk that the auditor will not detect a material misstatement.

disclosure. Process of divulging accounting information so that the content of financial statements is understood.

discount. Reduction from the full amount of a price or debt.

dividends. Distribution of earnings to owners of a corporation in cash, other assets of the corporation, or the corporation’s capital stock.

earnings per share (EPS). The amount of earnings attributable to each share of common stock. For convenience, the term is used to refer to either earnings or loss per share.

employee stock options plans. An employee stock ownership plan is an employee benefit plan that is described by the Employee Retirement Income Security Act of 1974 and the Internal Revenue Code of 1986 as a stock bonus plan, or combination stock bonus and money purchase pension plan, designed to invest primarily in employer stock. Also called an employee share ownership plan. Employee Stock Options Plans should not be confused with the term “ESOPs,” or Employee Stock Ownership Plans, which are retirement plans.

employee stock ownership plans (ESOPs). An employee stock ownership plan (ESOP) is a retirement plan in which the company contributes its stock to the plan for the benefit of the company’s employees. With an ESOP, you never buy or hold the stock directly. This type of plan should not be confused with employee stock options plans, which are not retirement plans. Instead, employee stock options plans give the employee the right to buy their company’s stock at a set price within a certain period of time.

equity. Residual interest in the assets of an entity that remains after deducting its liabilities. Also, the amount of a business’ total assets, less total liabilities. Also, the third section of a balance sheet, the other two being assets and liabilities.

equity security. Any security representing an ownership interest in an entity (for example, common, preferred, or other capital stock) or the right to acquire (for example, warrants, rights, and call options) or dispose of (for example, put options) an ownership interest in an entity at fixed or determinable prices. However, the term does not include convertible debt or preferred stock that by its terms either must be redeemed by the issuing entity or is redeemable at the option of the investor.

Page 90: Performing Preparation, Compilation, and Review Engagements

© 2021 Association of International Certified Professional Accountants. All rights reserved. 82

error. Act that departs from what should be done; imprudent deviation, unintentional mistake or omission.

executive compensation: Where to find in SEC reports. The federal securities laws require clear, concise and understandable disclosure about compensation paid to CEOs and certain other high-ranking executive officers of public companies. You can locate information about executive pay in (1) the company’s annual proxy statement; (2) the company’s annual report on Form 10-K; and (3) registration statements filed by the company to register securities for sale to the public.

expenditures. Expenditures to which capitalization rates are to be applied are capitalized expenditures (net of progress payment collections) for the qualifying asset that have required the payment of cash, the transfer of other assets, or the incurring of a liability on which interest is recognized (in contrast to liabilities, such as trade payables, accruals, and retainages on which interest is not recognized).

extraordinary items. Events and transactions distinguished by their unusual nature and by the infrequency of their occurrence. Extraordinary items are reported separately, less applicable income taxes, in the entity’s statement of income or operations.

Fair Disclosure, Regulation FD. On August 15, 2000, the SEC adopted Regulation FD to address the selective disclosure of information by companies and other issuers. Regulation FD provides that when an issuer discloses material nonpublic information to certain individuals or entities – generally, securities market professionals, such as stock analysts, or holders of the issuer’s securities who may well trade on the basis of the information – the issuer must make public disclosure of that information. In this way, the new rule aims to promote the full and fair disclosure.

fair market value. Price at which property would change hands between a buyer and a seller without any compulsion to buy or sell.

federal securities laws. The laws that govern the securities industry, include the Securities Act of 1933; Securities Exchange Act of 1934; Investment Company Act of 1940; Investment Advisers Act of 1940; and Public Utility Holding Company Act of 1935.

financial statements. Presentation of financial data including balance sheets, income statements and statements of cash flow, or any supporting statement that is intended to communicate an entity’s financial position at a point in time and its results of operations for a period then ended.

first in, first out (FIFO). Accounting method of valuing inventory under which the costs of the first goods acquired are the first costs charged to expense. Commonly known as FIFO.

fiscal year. Period of 12 consecutive months chosen by an entity as its accounting period which may or may not be a calendar year.

fixed asset. Any tangible asset with a life of more than one year used in an entity’s operations.

Page 91: Performing Preparation, Compilation, and Review Engagements

© 2021 Association of International Certified Professional Accountants. All rights reserved. 83

foreign currency translation. Restating foreign currency in equivalent dollars; unrealized gains or losses are postponed and carried in Stockholder’s Equity until the foreign operation is substantially liquidated.

Form 10-K. This is the report that most publicly traded companies file with the SEC on an annual basis. It provides a comprehensive overview of the company’s business and financial condition. Some companies choose to send their Form 10-K to their shareholders instead of sending a separate annual report. Currently, Form 10-K must be filed with the SEC within 90 days after the end of the company’s fiscal year.

Form 10-Q. The Form 10-Q is a report filed quarterly by most reporting companies. It includes unaudited financial statements and provides a continuing view of the company’s financial position during the year. The report must be filed for each of the first three fiscal quarters of the company’s fiscal year and is currently due within 45 days of the close of the quarter. In addition to Form 10-Q, companies provide annual reports to their shareholders and file Form 10-K on an annual basis with the SEC.

Form 8-K. This is the “current report” used to report material events or corporate changes that have previously not been reported by the company in a quarterly report (Form 10-Q) or annual report (Form 10-K).

Forms 3, 4, 5. Corporate insiders-meaning a company’s officers and directors, and any beneficial owners of more than 10% of a class of the company’s equity securities registered under Section 12 of the Securities Exchange Act of 1934 – must file with the SEC a statement of ownership regarding those securities. The initial filing is on Form 3. Changes in ownership are reported on Form 4. Insiders must file a Form 5 to report any transactions that should have been reported earlier on a Form 4 or were eligible for deferred reporting.

fraud. Willful misrepresentation by one person of a fact inflicting damage on another person.

gain. Excess of revenues received over costs relating to a specific transaction.

general ledger. Collection of all assets, liability, owners’ equity, revenue, and expense accounts.

generally accepted accounting principles (GAAP). Conventions, rules, and procedures necessary to define accepted accounting practice at a particular time. The highest level of such principles is set by the Financial Accounting Standards Board (FASB).

generally accepted auditing standards (GAAS). Standards set by the American Institute of Certified Public Accountants (AICPA) which concern the auditor’s professional qualities and judgment in the performance of his/her audit and in the actual report.

going concern. Assumption that a business can remain in operation long enough for all of its current plans to be carried out.

going private. A company “goes private” when it reduces the number of its shareholders to fewer than 300 and is no longer required to file reports with the SEC.

Page 92: Performing Preparation, Compilation, and Review Engagements

© 2021 Association of International Certified Professional Accountants. All rights reserved. 84

goodwill. An asset representing the future economic benefits arising from other assets acquired in a business combination or an acquisition by a not for profit entity that are not individually identified and separately recognized.

gross income. A tax term meaning all income from whatever source derived, except as otherwise provided in the income tax code.

guaranty. Legal arrangement involving a promise by one person to perform the obligations of a second person to a third person, in the event the second person fails to perform.

hedges. Protect an entity against the risk of adverse price or interest-rate movements on its assets, liabilities, or anticipated transactions. A hedge is used to avoid or reduce risks by creating a relationship by which losses on positions are counterbalanced by gains on separate positions in another market.

historical cost. The generally accepted method of accounting used in the primary financial statements that is based on measures of historical prices without restatement into units, each of which has the same general purchasing power.

income. Inflow of revenue during a period of time.

income statement. Summary of the effect of revenues and expenses over a period of time.

income tax basis. A special purpose framework that the reporting entity uses or expects to use to file its income tax return for the period covered by the financial statements.

initial public offerings (IPO). IPO stands for initial public offering and occurs when a company first sells its shares to the public.

initial public offerings, lockup agreements. Lockup agreements prohibit company insiders – including employees, their friends and family, and venture capitalists – from selling their shares for a set period of time. In other words, the shares are “locked up.” Before a company goes public, the company and its underwriter typically enter into a lockup agreement to ensure that shares owned by these insiders do not enter the public market too soon after the offering.

insider trading. “Insider trading” actually includes both legal and illegal conduct. The legal version is when corporate insiders – officers, directors, and employees – buy and sell stock in their own companies. Illegal insider trading refers generally to buying or selling a security, in breach of a fiduciary duty or other relationship of trust and confidence, while in possession of material, nonpublic information about the security. Insider trading violations may also include “tipping” such information, securities trading by the person “tipped,” and securities trading by those who misappropriate such information.

intangible asset. Asset having no physical existence such as trademarks and patents.

interest. Payment for the use or forbearance of money.

interim financial statements. Financial statements that report the operations of an entity for less than one year.

Page 93: Performing Preparation, Compilation, and Review Engagements

© 2021 Association of International Certified Professional Accountants. All rights reserved. 85

internal control. Process designed to provide reasonable assurance regarding achievement of various management objectives such as the reliability of financial reports.

inventory. Tangible property held for sale, or materials used in a production process to make a product.

investment. Expenditure used to purchase goods or services that could produce a return to the investor.

journal. Any book containing original entries of daily financial transactions.

last in, first out (LIFO). Accounting method of valuing inventory under which the costs of the last goods acquired are the first costs charged to expense. Commonly known as LIFO.

lease. Conveyance of land, buildings, equipment, or other assets from one person (Lessor) to another (Lessee) for a specific period of time for monetary or other consideration, usually in the form of rent.

leasehold. Property interest a lessee owns in the leased property.

ledger. Any book of accounts containing the summaries of debit and credit entries.

lessee. Person or entity that has the right to use property under the terms of a lease.

lessor. Owner of property, the temporary use of which is transferred to another (lessee) under the terms of a lease.

liability. Debts or obligations owed by one entity (Debtor) to another entity (Creditor) payable in money, goods, or services.

listing and delisting requirements. Before a company can begin trading on an exchange or the Nasdaq Stock Market, it must meet certain initial requirements or “listing standards.” The exchanges and the Nasdaq Stock Market set their own standards for listing and continuing to trade. The SEC does not set listing standards. The initial listing requirements mandate that a company meet specified minimum thresholds for the number of publicly traded shares, total market value, stock price, and number of shareholders. After a company starts trading, it must continue to meet different standards set by the exchanges or the Nasdaq Stock Market. Otherwise, the company can be delisted. These continuing standards usually are less stringent than the initial listing requirements.

long-term debt. Debt with a maturity of more than one year from the current date.

loss. Excess of expenditures over revenue for a period or activity. Also, for tax purposes, an excess of basis over the amount realized in a transaction.

lower of cost or market. Valuing assets for financial reporting purposes. Ordinarily, “cost” is the purchase price of the asset and “market” refers to its current replacement cost. Generally accepted accounting principles (GAAP) requires that certain assets (e.g., inventories) be carried at the lower of cost or market.

Page 94: Performing Preparation, Compilation, and Review Engagements

© 2021 Association of International Certified Professional Accountants. All rights reserved. 86

management discussion and analysis (MD&A). SEC requirement in financial reporting for an explanation by management of significant changes in operations, assets, and liquidity.

manipulation. Manipulation is intentional conduct designed to deceive investors by controlling or artificially affecting the market for a security. Manipulation can involve a number of techniques to affect the supply of, or demand for, a stock. They include spreading false or misleading information about a company; improperly limiting the number of publicly-available shares; or rigging quotes, prices, or trades to create a false or deceptive picture of the demand for a security.

marketable securities. Stocks and other negotiable instruments which can be easily bought and sold on either listed exchanges or over-the-counter markets.

mark-to-market. Method of valuing assets that results in adjustment of an asset’s carrying amount to its market value.

matching principle. The concept that all costs and expenses incurred in generating revenues must be recognized in the same reporting period as the related revenues.

materiality. Magnitude of an omission or misstatements of accounting information that, in the light of surrounding circumstances, makes it probable that the judgment of a reasonable person relying on the information would change or be influenced.

mergers. Mergers are business transactions involving the combination of two or more companies into a single entity. Most state laws require that mergers be approved by at least a majority of the company’s shareholders if the merger will have a significant impact on the company.

modified cash basis. A special purpose framework that begins with the cash basis method (see Cash basis) and applies modifications having substantial support, such as recording depreciation on fixed assets or accruing income taxes.

NASDAQ. Nasdaq stands for the National Association of Securities Dealers Automated Quotation System. Unlike the New York Stock Exchange where trades take place on an exchange, Nasdaq is an electronic stock market that uses a computerized system to provide brokers and dealers with price quotes. The National Association of Securities Dealers, Inc. owns and operates The Nasdaq Stock Market.

net assets. Excess of the value of securities owned, cash, receivables, and other assets over the liabilities of the company.

net income. Excess or deficit of total revenues and gains compared with total expenses and losses for an accounting period.

net sales. Sales at gross invoice amounts less any adjustments for returns, allowances, or discounts taken.

net worth. Similar to equity, the excess of assets over liabilities.

Page 95: Performing Preparation, Compilation, and Review Engagements

© 2021 Association of International Certified Professional Accountants. All rights reserved. 87

nonpublic entity. Any entity other than (a) one whose securities trade in a public market either on a stock exchange (domestic or foreign) or in the over-the-counter market, including securities quoted only locally or regionally; (b) one that makes a filing with a regulatory agency in preparation for the sale of any class of its securities in a public market; or (c) a subsidiary, corporate joint venture, or other entity controlled by an entity covered by (a) or (b).

no-par stock. Stock authorized to be issued but for which no par value is set in the articles of incorporation. A stated value is set by the board of directors on the issuance of this type of stock.

no-par value. Stock or bond that does not have a specific value indicated.

notional. Value assigned to assets or liabilities that is not based on cost or market (e.g., the value of a service not yet rendered).

objectivity. Emphasizing or expressing the nature of reality as it is apart from personal reflection or feelings; independence of mind.

paid in capital. Portion of the stockholders’ equity which was paid in by the stockholders, as opposed to capital arising from profitable operations.

par value. Amount per share set in the articles of incorporation of a corporation to be entered in the capital stocks account where it is left permanently and signifies a cushion of equity capital for the protection of creditors.

parent company. Company that has a controlling interest in the common stock of another.

predecessor accountant. An accountant who (a) has reported on the most recent compiled or reviewed financial statements or was engaged to perform but did not complete a compilation or review of the financial statements, and (b) has resigned, declined to stand for reappointment, or been notified that his or her services have been or may be terminated.

preferred stock. Type of capital stock that carries certain preferences over common stock, such as a prior claim on dividends and assets.

premium. (1) Excess amount paid for a bond over its face amount. (2) In insurance, the cost of specified coverage for a designated period of time.

prepaid expense. Cost incurred to acquire economically useful goods or services that are expected to be consumed in the revenue-earning process within the operating cycle.

prescribed form. Any standard preprinted form designed or adopted by the body to which it is to be submitted, for example, forms used by industry trade associations, credit agencies, banks, and governmental and regulatory bodies other than those concerned with the sale or trading of securities. A form designed or adopted by the entity whose financial statements are to be compiled is not considered to be a prescribed form.

present value. Current value of a given future cash flow stream, discounted at a given rate.

principal. Face amount of a security, exclusive of any premium or interest. The basis for interest computations.

Page 96: Performing Preparation, Compilation, and Review Engagements

© 2021 Association of International Certified Professional Accountants. All rights reserved. 88

proxy statement. The SEC requires that shareholders of a company whose securities are registered under Section 12 of the Securities Exchange Act of 1934 receive a proxy statement prior to a shareholder meeting, whether an annual or special meeting. The information contained in the statement must be filed with the SEC before soliciting a shareholder vote on the election of directors and the approval of other corporate action. Solicitations, whether by management or shareholders, must disclose all important facts about the issues on which shareholders are asked to vote.

purchase method of accounting. Accounting for a merger by adding the acquired company’s assets at the price paid for them to the acquiring company’s assets.

quiet period. The term “quiet period,” also referred to as the “waiting period,” is not defined under the federal securities laws. The quiet period extends from the time a company files a registration statement with the SEC until SEC staff declares the registration statement “effective.” During this period, the federal securities laws limit what information a company and related parties can release to the public. Rule 134 of the Securities Act of 1933 discusses these limitations.

ratio analysis. Comparison of actual or projected data for a particular company to other data for that company or industry in order to analyze trends or relationships.

real property. Land and improvements, including buildings and personal property that is permanently attached to the land or customarily transferred with the land.

receivables. Amounts of money due from customers or other debtors.

reconciliation. Comparison of two numbers to demonstrate the basis for the difference between them.

Registration Under the Securities Act of 1933. Often referred to as the “truth in securities” law, the Securities Act of 1933 has two basic objectives: (1) To require that investors receive financial and other significant information concerning securities being offered for public sale; and (2) To prohibit deceit, misrepresentations, and other fraud in the sale of securities. The SEC accomplishes these goals primarily by requiring that companies disclose important financial information through the registration of securities. This information enables investors, not the government, to make informed judgments about whether to purchase a company’s securities.

Regulation D offerings. Under the Securities Act of 1933, any offer to sell securities must either be registered with the SEC or meet an exemption. Regulation D (or Reg D) provides three exemptions from the registration requirements, allowing some smaller companies to offer and sell their securities without having to register the securities with the SEC.

regulatory basis. A special purpose framework that the reporting entity uses to comply with the requirements or financial reporting provisions of a governmental regulatory agency to whose jurisdiction the entity is subject. An example is a basis of accounting insurance companies use pursuant to the rules of a state insurance commission.

Page 97: Performing Preparation, Compilation, and Review Engagements

© 2021 Association of International Certified Professional Accountants. All rights reserved. 89

reissued report. A report issued subsequent to the date of the original report that bears the same date as the original report. A reissued report may need to be revised for the effects of specific events; in these circumstances, the report should be dual-dated with the original date and a separate date that applies to the effects of such events.

related party transaction. Business or other transaction between persons who do not have an arm’s-length relationship (e.g., a relationship with independent, competing interests). The most common is between family members or controlled entities. For tax purposes, these types of transactions are generally subject to a greater level of scrutiny.

research and development (R&D). Research is a planned activity aimed at discovery of new knowledge with the hope of developing new or improved products and services. Development is the translation of research findings into a plan or design of new or improved products and services.

retained earnings. Accumulated undistributed earnings of a company retained for future needs or for future distribution to its owners.

revenue recognition. Method of determining whether or not income has met the conditions of being earned and realized or is realizable.

revenues. Sales of products, merchandise, and services; and earnings from interest, dividend, rents.

review. Accounting service that provides some assurance as to the reliability of financial information. In a review, a certified public accountant (CPA) does not conduct an examination under generally accepted auditing standards (GAAS). Instead, the accountant performs inquiry and analytical procedures that provide the accountant with a reasonable basis for expressing limited assurance that there are no material modifications that should be made to the statements for them to be in conformity with GAAP or, if applicable, with a special purpose framework.

risk management. Process of identifying and monitoring business risks in a manner that offers a risk/return relationship that is acceptable to an entity’s operating philosophy.

security. Any kind of transferable certificate of ownership including equity securities and debt securities.

short-term. Current; ordinarily due within one year.

SSARS. Statements on Standards for Accounting And Review Services issued by the AICPA Accounting and Review Services Committee (ARSC).

start-up costs. (1) Costs, excluding acquisition costs, incurred to bring a new unit into production. (2) Costs incurred to begin a business.

statement of cash flows. A statement of cash flows is one of the basic financial statements that is required as part of a complete set of financial statements prepared in conformity with generally accepted accounting principles. It categorizes net cash provided or used during a

Page 98: Performing Preparation, Compilation, and Review Engagements

© 2021 Association of International Certified Professional Accountants. All rights reserved. 90

period as operating, investing and financing activities, and reconciles beginning and ending cash and cash equivalents.

statement of financial condition. Basic financial statement, usually accompanied by appropriate disclosures that describe the basis of accounting used in its preparation and presentation as of a specified date, the entity’s assets, liabilities, and the equity of its owners. Also known as balance sheet.

statutory basis. See Regulatory basis.

straight-line depreciation. Accounting method that reflects an equal amount of wear and tear during each period of an asset’s useful life. For instance, the annual straight-line depreciation of a $10,000 asset expected to last ten years is $1,000.

strike price. Price of a financial instrument at which conversion or exercise occurs.

submission of financial statements. Presenting to a client or third party’s financial statements that the accountant has prepared either manually or through the use of computer software.

subsequent event. Material event that occurs after the end of the accounting period and before the publication of an entity’s financial statements. Such events are disclosed in the notes to the financial statements.

successor accountant. An accountant who has been invited to make a proposal for an engagement to compile or review financial statements and is considering accepting the engagement or an accountant who has accepted such an engagement.

tangible asset. Assets having a physical existence, such as cash, land, buildings, machinery, or claims on property, investments or goods in process.

tax. Charge levied by a governmental unit on income, consumption, wealth, or other basis.

third party. All parties except for members of management who are knowledgeable about the nature of the procedures applied and the basis of accounting and assumptions used in the preparation of the financial statements.

trade date. Date when a security transaction is entered into, to be settled on at a later date. Transactions involving financial instruments are generally accounted for on the trade date.

treasury bill. Short-term obligation that bears no interest and is sold at a discount.

treasury bond. Long-term obligation that matures more than five years from issuance and bears interest.

treasury note. Intermediate-term obligation that matures one to five years from issuance and bears interest.

treasury stock. Stock reacquired by the issuing company. It may be held indefinitely, retired, issued upon exercise of stock options, or resold.

trial balance. A trial balance consists of a listing of all of the general ledger accounts and their corresponding debit or credit balances. Also, in a trial balance, no attempt is made to

Page 99: Performing Preparation, Compilation, and Review Engagements

© 2021 Association of International Certified Professional Accountants. All rights reserved. 91

establish a mathematical relationship among the assets, liabilities, equity, revenues, and expenses except that total debits equal total credits.

unearned income. Payments received for services which have not yet been performed.

updated report. A report issued by a continuing accountant that takes into consideration information that he/she becomes aware of during his/her current engagement and that re-expresses his/her previous conclusions or, depending on the circumstances, expresses different conclusions on the financial statements of a prior period as of the date of his/her current report.

valuation allowance. Method of lowering or raising an object’s current value by adjusting its acquisition cost to reflect its market value by use of a contra account.

variance. Deviation or difference between an estimated value and the actual value.

work in progress. Inventory account consisting of partially completed goods awaiting completion and transfer to finished inventory.

working capital. Excess of current assets over current liabilities.

working papers. (1) Records kept by the auditor of the procedures applied, the tests performed, the information obtained, and the pertinent conclusions reached in the course of the audit. (2) Any records developed by a certified public accountant (CPA) during an audit.

yield. Return on an investment an investor receives from dividends or interest expressed as a percentage of the cost of the security.

Page 100: Performing Preparation, Compilation, and Review Engagements

© 2021 Association of International Certified Professional Accountants. All rights reserved. 92

Page 101: Performing Preparation, Compilation, and Review Engagements

Users of this course material are encouraged to visit www.aicpa.org/CPESupplements

to access supplemental learning material reflecting recent developments that may be applicable to this course.

Supplemental materials will be made available quarterly.

Page 102: Performing Preparation, Compilation, and Review Engagements
Page 103: Performing Preparation, Compilation, and Review Engagements

Learn More

governmental and not-for-profit accounting, auditing, and updates; internal control and fraud; audits of employee benefit plans and 401(k) plans; individual and corporate tax updates; and a vast array of courses in other areas of accounting and auditing, controllership,

management, consulting, taxation, and more!

Get your CPE when and where you want Self-study learning options that include on-demand, webcasts, and text formats with

superior quality and a broad portfolio of topics, including bundled products like – CPExpress® online learning for immediate access to hundreds of one- to four-credit

hour online courses for just-in-time learning at a price that is right. Annual Webcast Pass offering live Q&A with experts and unlimited access to the

scheduled lineup, all at an incredible discount.

Staff training programs for audit, tax and preparation, compilation, and review. Certificate programs offering comprehensive curriculums developed by practicing experts to

build fundamental core competencies in specialized topics. National conferences presented by recognized experts. Affordable courses on-site at your organization – visit aicpalearning.org/on-site for

more information.

Seminars sponsored by your state society and led by top instructors. For a complete list, visit aicpalearning.org/publicseminar.

Take control of your career development The Association's Competency and Learning website at https://competency.aicpa.org brings together a variety of learning resources and a self-assessment tool, enabling tracking and reporting of progress toward learning goals.

Visit www.AICPAStore.com to browse our CPEselections.

Continuing Professional EducationThank you for selecting the Association of International Certified Professional Accountants asyour continuing professional education provider. We have a diverse offering of CPE courses to help you expand your skill set and develop your competencies. Choose from hundreds of different titles spanning the major subject matter areas relevant to CPAs and CGMAs, including

cpaschal
Sticky Note
Marked set by cpaschal
Page 104: Performing Preparation, Compilation, and Review Engagements

CPExpress® online learning CPExpress® from the Association of International Certified Professional Accountants combines quality learning with easy online access to deliver convenient, flexible, and affordable learning that fits your busy schedule. Reach your professional development goals with immediate access to hundreds of one- to four-credit hour CPE courses designed to inform, educate, and reinforce the core competencies of the accounting profession. With CPExpress, you

• have unlimited access to hundreds of self-study courses and 500+ CPE hours of training, includingregular updates to content throughout the year;

• can customize your learning experience with relevant professional development topics, includingtaxes, auditing, accounting, finance, information technology, and more;

• get insights on the latest standards and trends in the accounting profession; and• earn your credits fast with 24/7 easy online access to CPE courses.

Group pricingIf you have 5 or more people, contact one of our learning consultants regarding group discounts that can help maximize the ROI of your learning initiatives. Learn more at aicpastore.com/content/media/Training/group-training.jsp or call 800.634.6780, option 1.

Subscribe today at AICPAStore.com/CPExpressor call 888.777.7077.

Annual Webcast PassWith our Annual Webcast Pass, you can explore a variety of topics specific to what you’re interested in now — or what you’d like to try in the future. You can choose courses from outside your required sub-ject area and expand your knowledge at no additional cost. It’s professional guidance, year-round. You’ll get

• professional guidance from top experts, regulators and industry leaders across major fields of study,including auditing, accounting, taxes, information technology, and more;

• learning content that encourages you to look closely at your own practice, ask questions; and makeconnections;

• live webcasts from one to eight hours, so you can plan your learning around your schedule;• one year of access to more than 500 AICPA webcasts on hot topics (such as tax reform, blockchain,

data analytics, and more), critical updates, and the latest standards.Group pricingIf you have 5 or more people, contact one of our learning consultants regarding group discounts that can help maximize the ROI of your learning initiatives. Learn more at aicpastore.com/content/media/Training/group-training.jsp or call 800.634.6780, option 1.

Subscribe today at AICPAStore.com/CPExpressor call 888.777.7077.

Page 105: Performing Preparation, Compilation, and Review Engagements

Your strategic learning partner Let us help prepare your staff for the future.

AICPA Learning resources can help you:• Create a learning culture to attract and retain talent• Enrich staff competency and stay current on

changing regulations• Sharpen your competitive edge• Capitalize on emerging opportunities• Meet your goals and positively impact your

bottom line• Address CPE/CPD compliance

Flexible learning options include:• On-site training• Conferences• Webcasts• Certificate programs• Online self-study• Publications

© 2017 Association of International Certified Professional Accountants. All rights reserved. AICPA and American Institute of CPAs are trademarks of the American Institute of Certified Public Accountants and are registered in the United States, European Union and other countries. The Globe Design is a trademark owned by the Association of International Certified Professional Accountants and licensed to the AICPA. 23535-CAP

An investment in learning can directly impact your bottom line. Contact an AICPA learning consultant to begin your professional development planning.

Call: 800.634.6780, option 1 Email: [email protected]

What is your current approach to learning? One size does not fit all. Your organization is unique, and your approach to learning and competency should be, too. But where do you start? Choose a strategic partner to help you assess competencies and gaps, design a customized learning plan, and measure and maximize the ROI of your learning and development initiatives.

We offer a wide variety of learning programs for finance professionals at every stage of their career.

Page 106: Performing Preparation, Compilation, and Review Engagements

AICPAStore.com | CIMAglobal.com

1811-9880 Update CPE Text Covers w/ AICPA&CIMA logo.indd 1-3